Sei sulla pagina 1di 34

CONFLICT OF LAWS iv) Djumantan vs Domingo b) Marcos vs Comelec (1995)

PRIVATE INTERNATIONAL LAW (1995) DONE c) Domino vs Comelec (1999)


v) Oh Hek How vs Republic d) Jao vs CA (2002) VINA ARANETA
MIDTERMS (1969) DONE e) Romualdez vs RTC (1993)
vi) RP vs Sayo (1990) EUNICE f) Romualdez-Marcos vs Comelec (1995)
DONE g) Jalosjos vs Comelec (2012)
MEMBERS:
1. Alano, Dave Abby vii) RP vs Valencia (1986) - h) Caasi vs Comelec (1990)
DONE i) San Luis vs San Luis (2007)
2. Ambrocio, Eunice
b) Lost GRETCHEN
3. Alaba, Mishing
4. Baucan, Kareen i) Board vs Callano (1968)
DONE X. The Problem of Renvoi
5. Cagampang, Vina
6. Canedo, Gretchen ii) Yu vs Defensor Santiago 1) Renvoi, Definition
(1989) DONE 2) How does the problem of Renvoi arise?
7. Laurente, Kaye
iii) Coquilla vs Comelec (2002) 3) Transmission
VI. Characterization of Conflict Rules (No Case) DONE a) Anzar vs Christensen-Garcia (1963)
iv) Valles vs Comelec (2000) b) Bellis vs Bellis (1967)
VII. Personal Status and Capacity (No Case) MISHING DONE c) PCIB vs Escolin (1974)
c) Reacquired d) Gibbs vs Government (1933)
VIII. Nationality Theory i) RP vs Dela Rosa (1994) e) Miciano vs Brimo (1924) KAYE
DONE LAURENTE
1) What is the nationality theory?
a) Ellis vs. RP, April 30, 1963 DONE ii) Labo vs Comelec (1989)
DONE
2) Nationality vs Citizenship
3) Determination of Nationality iii) Labo vs Comelec (1992)
DONE
a) Board of Immigration vs Callano GR L-
24530 DONE iv) Frivaldo vs Comelec (1996)
4) Defects of the Nationality Theory v) Petition, Benjami Dacaycay
(2007) KAE BAUCAN
5) Three (3) Kinds of Citizens of the Philippines
a) Anzar vs Comelec (1990) DONE 8) Statelessness
b) AASJS vs Datumanong (2007) DONE a) Kookooritchion vs SolGen (1948)
6) Two theories on Whether place or ancestry
IX. Domiciliary Theory
determines citizenship
7) How citizenship is acquired/lost/reacquired 1) What is the Domiciliary Theory?
a) Acquired 2) Defects of the theory
i) Ngo Burca vs RP (1967) 3) Citizenship vs Domicile
DAVE DONE 4) Residence vs Domicile
ii) Mo Ya Lim Yao vs 5) Definition of Domicile
Commissioner (1971) DONE 6) Three kinds of Domicile
iii) Lee vs Commissioner (1971) 7) Rules for Domicile of Choice
DONE 8) Fundamental Principles Governing Domicile of
Choice
a) Aquino vs Comelec (1995)

Conflict of Laws (Private International Law) by Atty. Kristine Mae M. Quibod, EnP
JMC College of Law A.Y. 2017-2018 | Case Digest Compilation by: #TeamDreamworks Page 1
What is the nationality theory? under either the nationality theory or the domiciliary
Ellis vs. RP, April 30, 1963 Issue: Whether or not being permanent residents in the theory.
Philippines, petitioners are qualified to adopt Baby Rose?
G.R. No. L-16922 April 30, 1963
Held: NO.
IN RE: ADOPTION OF CHILD BAPTIZED UNDER THE Article 335 of the Civil Code of the Philippines, provides
NAME OF ROSE, that:
MARVIN G. ELLIS and GLORIA C. ELLIS, petitioners-
appellees, "The following cannot adopt:
vs. *******
REPUBLIC OF THE PHILIPPINES, oppositor-
appellant. "(4) Non-resident aliens;"
*******
Overview: This is an appeal taken by the Government
from a decision of the Court of First Instance of This legal provision is too clear to require interpretation.
Pampanga granting the petition of Marvin G. Ellis and No matter how much we may sympathize with the plight
Gloria C. Ellis for the adoption of a Filipino baby girl of Baby Rose and with the good intentions of petitioners
named Rose. herein, the law leaves us no choice but to apply its
explicit terms, which unqualifiedly deny to petitioners the
Facts: Petitioner Marvin G. Ellis, a native of San power to adopt anybody in the Philippines.
Francisco, California, is 28 years of age. He married
Gloria C. Ellis in Banger, Maine, United States. Both of In this connection, it should be noted that this is a
them are citizens of the United States. proceedings in rem, which no court may entertain unless
it has jurisdiction, not only over the subject matter of the
Baby Rose was born on September 26, 1959 at the case and over the parties, but also over the res, which is
Caloocan Maternity Hospital. A few days later, the the personal status of Baby Rose as well as that of
mother of Rose left her with the Heart of Mary Villa petitioners herein. Our Civil Code (Art. 15) adheres to the
institution for unwed mothers and their babies—stating theory that jurisdiction over the status of a natural person
that she (the mother) could not take care of Rose without is determined by the latters' nationality. Pursuant to this
bringing disgrace upon her (the mother's) family. theory, we have jurisdiction over the status of Baby Rose,
she being a citizen of the Philippines, but not over the
Hence, Mr. and Mrs. Ellis filed a petition with the Court of status of the petitioners, who are foreigners.
First Instance of Pampanga for the adoption of the
aforementioned baby. Under our political law, which is patterned after the
Anglo-American legal system, we have, likewise, adopted
At the time of the hearing of the petition on January 14, the latter's view to the effect that personal status, in
I960, petitioner Marvin G. Ellis and his wife had been in general, is determined by and/or subject to the
the Philippines for three (3) years, he being assigned jurisdiction of the domiciliary law.
thereto as staff sergeant in the United States Air Force
Base, in Angeles, Pampanga, where both lived at that Inasmuch as petitioners herein are not domiciled in the
time. They had been in the Philippines before, or, to be Philippines — and, hence, non-resident aliens - we
exact, in 1953. cannot assume and exercise jurisdiction over the status,

Conflict of Laws (Private International Law) by Atty. Kristine Mae M. Quibod, EnP
JMC College of Law A.Y. 2017-2018 | Case Digest Compilation by: #TeamDreamworks Page 2
Nationality vs Citizenship — a Chinese citizen," and concluding that "until the even renunciation cannot be cited in support of the
Determination of Nationality petitioners (Go-Callano) left for China in 1947, they must conclusion that petition lost their Philippine citizenship
Board of Immigration vs Callano GR L-24530 be considered as citizens of the Philippines as they were because the law requires an express renunciation which
born of a Filipino mother and an alien father who, means a renunciation that is made known distinctly and
G.R. No. L-24530 October 31, 1968 however, was not married to their mother." explicitly and not left to inference or implication; a
renunciation manifested by direct and appropriate
BOARD OF IMMIGRATION COMMISSIONERS and Court of Appeals: It found that herein respondents were language, as distinguished from that which is inferred
COMMISSIONER OF IMMIGRATION, petitioners, the illegitimate children of Go Chiao Lin, a Chinese from conduct.
vs. citizen, and Emilia Callano, a Filipino citizen. It reversed
BEATO GO CALLANO, MANUEL GO CALLANO, the decision of the lower court. Moreover, herein petitioners were all minors when they
GONZALO GO CALLANO, JULIO GO CALLANO and where brought to China in 1446. They were without legal
THE COURT OF APPEALS, respondents. Issue: Whether or not respondent Go-Callano brothers capacity to renounce their status. Upon their return to the
are Filipino Citizens? Philippines only Beato Go Callano had attained the age
Facts: On July 13, 1962, the Department of Foreign of majority, but even as to him there could not have been
Affairs informed the Commissioner of Immigration that, Held: YES. The following portions of the decision of the renunciation because he did not manifest by direct and
on the basis of the findings made by the National Bureau Court of Appeals would seem to be sufficient answer: appropriate language that he was disclaiming Philippine
of Investigation, the signatures of former Secretary of citizenship. On the contrary, after he has attained the age
Foreign Affairs, Felixberto M. Serrano, on certain The petitioners are admittedly Filipino citizens at of majority, he applied for registration as a Philippine
documents, amongst them authorizing the documentation birth, and their status must be governed by Philippine law citizen and sought entry into this country, which are clear
of Beato Go Callano and others, were not authentic. wherever they may be, in conformity with Article 15 indicia of his intent to continue his former status. The
(formerly Article 9) of the Civil Code which provides as foregoing shows that the petitioners have not lost their
The Commissioner of Immigration issued a warrant of follows: "Laws relating to family rights and duties, or to Philippine citizenship.
exclusion commanding the deportation officer "to carry the status, conditions and legal capacity of persons are
out the exclusion of the above-named applicants (the Go binding upon citizens of the Philippines, even though
Callano brothers) on the first available transportation and living abroad." Under Article IV, Section 2, of the
on the same class of accommodation in which they Philippine Constitution, "Philippine citizenship may be lost
arrived to the port whence they came or to the country of or reacquired in the manner provided by law," which
which they are nationals. implies that the question of whether a Filipino has lost his
Philippine citizenship shall be determined by no other
The warrant of exclusion, for one reason or another, was than the Philippine law.
not served immediately upon the parties ordered
deported, who, on November 16, 1962, filed in the Court Issue (2): Whether or not respondents renounced their
of First Instance of Manila an action for injunction to Filipino citizenship when they recognized their father?
restrain the Board of Immigration Commissioners and the
Commissioner Of Immigration from executing the order. Held (2): NO. Renunciation must be express.
They based their prayer for injunction on the ground that
they were not aliens, but Filipino citizens. Recognition of the petitioners by their alien father is not
among the ground for losing Philippine citizenship under
CFI: The CFI held that the Go-Callano brothers were Philippine law, and it cannot be said that the petitioners
citizens of the Republic of China. It found that "the lost their former status by reason of such recognition.
petitioners herein are the illegitimate children of Emilia About the only mode of losing Philippine citizenship
Callano, a Filipino citizen, with her common-law husband which closely bears on the petitioners is renunciation. But

Conflict of Laws (Private International Law) by Atty. Kristine Mae M. Quibod, EnP
JMC College of Law A.Y. 2017-2018 | Case Digest Compilation by: #TeamDreamworks Page 3
Defects of the Nationality Theory six months; and that he has been a registered voter in foregoing, the respondent is an American and "being an
Three (3) Kinds of Citizens of the Philippines the Philippines since 1965. American", private respondent "must have taken and
Anzar vs Comelec (1990) sworn to the Oath of Allegiance required by the U.S.
On March 3, 1988, COMELEC directed the Board of Naturalization Laws."
G.R. No. 83820 May 25, 1990 Canvassers to proclaim the winning candidates. Having
obtained the highest number of votes, Osmena was Philippine courts are only allowed to determine who are
JOSE B. AZNAR (as Provincial Chairman of PDP proclaimed the Provincial Governor of Cebu. On June 11, Filipino citizens and who are not. Whether or not a
Laban in Cebu), petitioner, 1988, COMELEC dismissed the petition for person is considered an American under the laws of the
vs. disqualification for not having been timely filed and for United States does not concern Us here.
COMMISSION ON ELECTIONS and EMILIO MARIO lack of sufficient proof that Osmena is not a Filipino
RENNER OSMEÑA, respondents. citizen. By virtue of his being the son of a Filipino father, the
presumption that Osmena is a Filipino remains. It was
Facts: On November 19, 1987, private respondent Emilio Issue: Whether or not Respondent Osmena is a Filipino incumbent upon Aznar to prove that Osmena had lost his
"Lito" Osmeña filed his certificate of candidacy with the Citizen? Philippine citizenship however, he failed to positively
COMELEC for the position of Provincial Governor of establish this fact.
Cebu Province in the January 18, 1988 local elections. Held: YES. Petitioner's contention that private
On January 22, 1988, the Cebu PDP-Laban Provincial respondent is not a Filipino citizen and, therefore, Osmena vehemently denies having taken the oath of
Council (Cebu-PDP Laban, for short), as represented by disqualified from running for and being elected to the allegiance of the US. He is a holder of a valid and
petitioner Jose B. Aznar in his capacity as its incumbent office of Provincial Governor of Cebu, is not supported subsisting Philippine passport and has continuously
Provincial Chairman, filed with the COMELEC a petition by substantial and convincing evidence. participated in the electoral process in this country since
for the disqualification of Osmeña on the ground that he 1963 up to the present, both as a voter and as a
is allegedly not a Filipino citizen, being a citizen of the In the proceedings before the COMELEC, the petitioner candidate. Thus, Osmena remains a Filipino and the loss
United States of America. failed to present direct proof that private respondent had of his Philippine citizenship cannot be presumed.
lost his Filipino citizenship by any of the modes provided
On January 28, 1988, the COMELEC en banc resolved for under C.A. No. 63. Among others, these are:
to order the Board to continue canvassing but to suspend
the proclamation. At the hearing before the COMELEC, (1) by naturalization in a foreign country;
Aznar presented the following exhibits tending to show (2) by express renunciation of citizenship;
that Osmena is an American citizen: Application for Alien and
(3) by subscribing to an oath of allegiance to
Registration of the Bureau of Immigration dated
support the Constitution or laws of a foreign country.
November 21, 1979; Alien Certificate of Registration From the evidence, it is clear that private respondent
dated November 21, 1979; Permit to Re-enter the Osmeña did not lose his Philippine citizenship by any
Philippines dated November 21, 1979; Immigration of the three mentioned hereinabove or by any other
Certificate of Clearance dated January 3, 1980. Osmeña mode of losing Philippine citizenship.
on the other hand, maintained that he is a Filipino citizen,
alleging: that he is the legitimate child of Dr. Emilio D. In concluding that private respondent had been
Osmeña, a Filipino and son of the late President Sergio naturalized as a citizen of the United States of America,
Osmeña, Sr.; that he is a holder of a valid and subsisting the petitioner merely relied on the fact that private
Philippine Passport issued on March 25, 1987; that he respondent was issued alien certificate of registration and
has been continuously residing in the Philippines since was given clearance and permit to re-enter the
birth and has not gone out of the country for more than Philippines by the Commission on Immigration and
Deportation. Petitioner assumed that because of the

Conflict of Laws (Private International Law) by Atty. Kristine Mae M. Quibod, EnP
JMC College of Law A.Y. 2017-2018 | Case Digest Compilation by: #TeamDreamworks Page 4
AASJS vs Datumanong (2007) country shall be deemed not to have lost their Philippine What Rep. Act No. 9225 does is allow dual citizenship to
citizenship." The OSG further claims that the oath in natural-born Filipino citizens who have lost Philippine
G.R. No. 160869 May 11, 2007 Section 3 does not allow dual allegiance since the oath citizenship by reason of their naturalization as citizens of
taken by the former Filipino citizen is an effective a foreign country.
AASJS (ADVOCATES AND ADHERENTS OF SOCIAL renunciation and repudiation of his foreign citizenship.
JUSTICE FOR SCHOOL TEACHERS AND ALLIED The fact that the applicant taking the oath recognizes and On its face, it does not recognize dual allegiance. By
WORKERS) MEMBER - HECTOR GUMANGAN accepts the supreme authority of the Philippines is an swearing to the supreme authority of the Republic, the
CALILUNG, Petitioner, unmistakable and categorical affirmation of his undivided person implicitly renounces his foreign citizenship.
vs. loyalty to the Republic. Plainly, from Section 3, Rep. Act No. 9225 stayed clear
THE HONORABLE SIMEON DATUMANONG, in his out of the problem of dual allegiance and shifted the
official capacity as the Secretary of Justice, Issue: Whether or not the assailed law is burden of confronting the issue of whether or not there is
Respondent. unconstitutional? dual allegiance to the concerned foreign country. What
happens to the other citizenship was not made a concern
Facts: Petitioner filed a petition for prohibition to prevent Held: NO. In resolving the aforecited issues in this of Rep. Act No. 9225.
Justice Secretary Datumanong from implementing R. A. case, resort to the deliberations of Congress is
9225 entitled "An Act Making the Citizenship of Philippine necessary to determine the intent of the legislative
Citizens Who Acquire Foreign Citizenship Permanent, branch in drafting the assailed law. During the
Amending for the Purpose Commonwealth Act No. 63, deliberations, the issue of whether Rep. Act No. 9225
As Amended, and for Other Purposes." which was signed would allow dual allegiance had in fact been the
into law by President Gloria M. Arroyo on August 29, subject of debate.
2003.
X x x Rep. Locsin underscored that the
Petitioner argued that R.A. 9225 is unconstitutional as it measure does not seek to address the constitutional
violates Sec. 5, Article VI of the Constitution which states injunction on dual allegiance as inimical to public
interest. He said that the proposed law aims to
that “dual allegiance of citizens is inimical to national
facilitate the reacquisition of Philippine citizenship by
interest and shall be dealt with by law.” speedy means. However, he said that in one sense, it
addresses the problem of dual citizenship by requiring
Petitioner contends that RA 9225 cheapens Philippine the taking of an oath. He explained that the problem of
citizenship. He avers that Sections 2 and 3 thereof, dual citizenship is transferred from the Philippines to
together, allow dual allegiance and not dual citizenship. the foreign country because the latest oath that will be
Petitioner maintains that Section 2 allows all Filipinos, taken by the former Filipino is one of allegiance to the
either natural-born or naturalized, who become foreign Philippines and not to the United States, as the case
may be. He added that this is a matter which the
citizens, to retain their Philippine citizenship without
Philippine government will have no concern and
losing their foreign citizenship. Section 3 permits dual competence over. X x x
allegiance because said law allows natural-born citizens
of the Philippines to regain their Philippine citizenship by It is clear that the intent of the legislature in drafting Rep.
simply taking an oath of allegiance without forfeiting their Act No. 9225 is to do away with the provision in
foreign allegiance. Commonwealth Act No. 635 which takes away Philippine
citizenship from natural-born Filipinos who become
The Office of the Solicitor General (OSG) claims that naturalized citizens of other countries.
Section 2 merely declares as a state policy that
"Philippine citizens who become citizens of another

Conflict of Laws (Private International Law) by Atty. Kristine Mae M. Quibod, EnP
JMC College of Law A.Y. 2017-2018 | Case Digest Compilation by: #TeamDreamworks Page 5
Two theories on Whether place or ancestry Held: NO. By constitutional and legal precepts, an alien
determines citizenship woman who marries a Filipino citizen, does not — by the The reason for exacting recital in the petition of present
How citizenship is acquired/lost/reacquired mere fact of marriage - automatically become a Filipino and former places of residence is that "information
Acquired - Ngo Burca vs RP (1967) citizen. regarding petitioner and objection to his application are
apt to be provided by people in his actual, physical
G.R. No. L-24252 January 30, 1967 Thus, by Article IV of the Constitution, citizenship is surrounding". the State is deprived of full opportunity to
limited to: make inquiries as to petitioner's fitness to become a
IN RE petition to declare ZITA NGO to possess all citizen, if all the places of residence do not appear in the
(1) Those who are citizens of the Philippine Islands at
qualifications and none of the disqualifications for petition. So it is, that failure to allege a former place of
the time of the adoption of this Constitution.
naturalization under Commonwealth Act 473 for the residence is fatal.
purpose of cancelling her alien registry with the (2) Those born in the Philippine Islands of foreign
BUREAU OF IMMIGRATION. parents who, before the adoption of this Constitution, We find one other flaw in petitioner's petition. Said
ZITA NGO BURCA, petitioner and appellee, had been elected to public office in the Philippine petition is not supported by the affidavit of at least two
vs. Islands. credible persons, "stating that they are citizens of the
REPUBLIC OF THE PHILIPPINES, oppositor and Philippines and personally know the petitioner to be a
appellant. (3) Those whose fathers are citizens of the resident of the Philippines for the period of time required
Philippines.
by this Act and a person of good repute and morally
Facts: Petitioner filed with the Court of First Instance of irreproachable, and that said petitioner has in their
(4) Those whose mothers are citizens of the
Leyte a petition alleging that she is married to Filipino Philippines and, upon reaching the age of majority, opinion all the qualifications necessary to become a
citizen and possesses all the qualifications and none the elect Philippine citizenship. citizen of the Philippines and is not in any way
disqualifications for naturalization under Commonwealth disqualified under the provisions of this Act.
Act 473 and praying that a declaration to such effect be (5) Those who are naturalized in accordance with law.
made by the Court for the purpose of laying the basis for Petitioner likewise failed to "set forth the names and post-
the cancellation by the Bureau of Immigration of her alien Jurisprudence has since stabilized the import of the office addresses of such witnesses as the petitioner may
certificate of registration. constitutional and statutory precepts just quoted with a desire to introduce at the hearing of the case". These
uniform pronouncement that an alien wife of a Filipino witnesses should indeed prove in court that they are
The Solicitor General opposed and moved to dismiss the citizen may not acquire the status of a citizen of the reliable insurers of the character of petitioner. Short of
petition on two main grounds, viz: (1) that "there is no Philippines unless there is proof that she herself may be this, the petition must fail.
proceeding established by law, or the rules for the judicial lawfully naturalized
declaration of the citizenship of an individual"; and (2) Here, the case was submitted solely on the testimony of
that as an application for Philippine citizenship, "the Indeed, the political privilege of citizenship should not to the petitioner. No other witnesses were presented. This
petition is fatally defective for failure to contain or mention any alien woman on the sole basis of her marriage to a does not meet with the legal requirement. Upon the view
the essential allegations required under Section 7 of the Filipino — "irrespective of moral character, ideological we take of his case, the judgment appealed from is
Naturalization Law", such as, among others, petitioner's beliefs, and identification with Filipino ideals, customs hereby reversed and the petition dismissed.
former places of residence, and the absence of the and traditions".
affidavits of at least two supporting witnesses.
On the merits of the case (The SC treated the petition
The judgement appealed therefrom declared that Burca as that of a petition for naturalization) Further discussion on Citizenship
was indeed a Filipino citizen.
Section 7 of the Naturalization Law requires that a There is no law or rule which authorizes a declaration of Filipino
Issue: Whether or not Burca is a Filipino Citizen? petition for naturalization should state petitioner's citizenship. Citizenship is not an appropriate subject for
"present and former places of residence. declaratory judgment proceedings. And in one case, we held

Conflict of Laws (Private International Law) by Atty. Kristine Mae M. Quibod, EnP
JMC College of Law A.Y. 2017-2018 | Case Digest Compilation by: #TeamDreamworks Page 6
that citizenship of an alien woman married to a Filipino must be immediately preceding the filing of the petition; and (3) Any
determined in an "appropriate proceeding". action by any other office, agency, board or official,
administrative or otherwise — other than the judgment of a
Speculations arise as to the import of the term "appropriate competent court of justice — certifying or declaring that an alien
proceeding". The record of this case disclose that, in some wife of the Filipino citizen is also a Filipino citizen, is hereby
quarters, opinion is advanced that the determination of whether declared null and void.
an alien woman married to a Filipino shall be deemed a Filipino
citizen, may be made by the Commissioner of Immigration. 11
Conceivably, absence of clear legal direction on the matter could
have given rise to divergence of views. We should aim at drying
up sources of doubt. Parties interested should not be enmeshed
in jurisdictional entanglements. Public policy and sound practice,
therefore, suggest that a clear-cut ruling be made on this
subject.

If an alien woman married to a Filipino does not become ipso


facto a citizen, then she must have to file a "petition for
citizenship" in order that she may acquire the status of a Filipino
citizen. Authority for this view is Section 7 of the Revised
Naturalization Law in which the plain language is: "Any person
desiring to acquire Philippine citizenship, shall file with the
competent court" a petition for the purpose. And this, because
such alien woman is not a citizen, and she desires to acquire it.
The proper forum, Section 8 of the same law points out, is the
Court of First Instance of the province where the petitioner has
resided "at least one year immediately preceding the filing of the
petition".

It is quite plain that the determination of whether said alien wife


should be given the status of a citizen should fall within the area
allocated to competent courts. That this is so, is exemplified by
the fact that this Court has taken jurisdiction in one such case
originating from the court of first instance, where an alien woman
had directly sought naturalization in her favor. 12

And, as nothing in the Revised Naturalization Law empowers


any other office, agency, board or official, to determine such
question, we are persuaded to say that resolution thereof rests
exclusively with the competent courts.

We accordingly rule that: (1) An alien woman married to a


Filipino who desires to be a citizen of this country must apply
therefor by filing a petition for citizenship reciting that she
possesses all the qualifications set forth in Section 2, and none
of the disqualifications under Section 4, both of the Revised
Naturalization Law; (2) Said petition must be filed in the Court of
First Instance where petitioner has resided at least one year

Conflict of Laws (Private International Law) by Atty. Kristine Mae M. Quibod, EnP
JMC College of Law A.Y. 2017-2018 | Case Digest Compilation by: #TeamDreamworks Page 7
No. L-21289. October 4, 1971. At the hearing which took place one and a half years no proof that the citizenship is not vested as of the date
MOY YA LIM YAO alias EDILBERTO AGUINALDO LIM after her arrival, it was admitted that Lau Yuen Yeung of marriage or the husband's acquisition of citizenship, as
and LAU YUEN YEUNG, petitioners-appellants vs. could not write and speak either English or Tagalog, the case may be, for the truth is that the situation obtains
THE COMMISSIONER OF IMMIGRATION, respondent- except for a few words. She could not name any Filipino even as to native-born Filipinos. Everytime the citizenship
appellee. neighbor, with a Filipino name except one, Rosa. She did of a person is material or indispensible in a judicial or
not know the names of her brothers-in-law, or sisters-in- administrative case. Whatever the corresponding court or
Doctrine: law. As a result, the Court of First Instance of Manila administrative authority decides therein as to such
Alien woman who marries a Filipino citizen ipso facto denied the prayer for preliminary injunction. citizenship is generally not considered as res judicata,
becomes a Filipina provided she is not disqualified to be hence it has to be threshed out again and again as the
a citizen of the Philippines under section 4 of Issue: occasion may demand. Lau Yuen Yeung, was declared
Commonwealth Act 473. Whether Lau Yuen Yeung ipso facto became a Filipino to have become a Filipino citizen from and by virtue of
citizen upon her marriage to a Filipino citizen. her marriage to Moy Ya Lim Yao al as Edilberto
Facts: Aguinaldo Lim, a Filipino citizen of 25 January 1962.
On February 8, 1961, Lau Yuen Yeung applied for a Ruling:
passport visa to enter the Philippines as a non-immigrant. Yes. Under Section 15 of Commonwealth Act 473, an
In the interrogation made in connection with her alien woman marrying a Filipino, native born or
application for a temporary visitor's visa to enter the naturalized, becomes ipso facto a Filipina provided she is
Philippines, she stated that she was a Chinese residing not disqualified to be a citizen of the Philippines under
at Kowloon, Hong Kong, and that she desired to take a Section 4 of the same law. Likewise, an alien woman
pleasure trip to the Philippines to visit her great grand married to an alien who is subsequently naturalized here
uncle, Lau Ching Ping. She was permitted to come into follows the Philippine citizenship of her husband the
the Philippines on 13 March 1961 for a period of one moment he takes his oath as Filipino citizen, provided
month. that she does not suffer from any of the disqualifications
under said Section 4. Whether the alien woman requires
On the date of her arrival, Asher Y. Cheng filed a bond in to undergo the naturalization proceedings, Section 15 is
the amount of P1,000.00 to undertake, among others, a parallel provision to Section 16. Thus, if the widow of
that said Lau Yuen Yeung would actually depart from the an applicant for naturalization as Filipino, who dies during
Philippines on or before the expiration of her authorized the proceedings, is not required to go through a
period of stay in this country or within the period as in his naturalization proceedings, in order to be considered as a
discretion the Commissioner of Immigration or his Filipino citizen hereof, it should follow that the wife of a
authorized representative might properly allow. living Filipino cannot be denied the same privilege.

After repeated extensions, Lau Yuen Yeung was allowed This is plain common sense and there is absolutely no
to stay in the Philippines up to 13 February 1962. On 25 evidence that the Legislature intended to treat them
January 1962, she contracted marriage with Moy Ya Lim differently. As the laws of our country, both substantive
Yao alias Edilberto Aguinaldo Lim an alleged Filipino and procedural, stand today, there is no such procedure
citizen. Because of the contemplated action of the (a substitute for naturalization proceeding to enable the
Commissioner of Immigration to confiscate her bond and alien wife of a Philippine citizen to have the matter of her
order her arrest and immediate deportation, after the own citizenship settled and established so that she may
expiration of her authorized stay, she brought an action not have to be called upon to prove it everytime she has
for injunction. to perform an act or enter into a transaction or business
or exercise a right reserved only to Filipinos), but such is

Conflict of Laws (Private International Law) by Atty. Kristine Mae M. Quibod, EnP
JMC College of Law A.Y. 2017-2018 | Case Digest Compilation by: #TeamDreamworks Page 8
No. L-23446. December 20, 1971. She, therefore, prayed that “after due notice and hearing exercise a right reserved only to Filipinos? The ready
FELISA LEE alias LEESIU LUAN, assisted by her as provided by law the petitioner be adjudged to have answer to such question is that as the laws of our
husband, JACKSON BARRA, petitioners-appellees, acquired the citizenship of her husband, Jackson Barra, country, both substantive and procedural, stand
vs.COMMISSIONER OF IMMIGRATION,respondent- who is a Filipino and to order the respondent to cancel today, there is no such procedure, but such paucity
appellant. her alien papers.” is no proof that the citizenship under discussion is
not vested as of the date of marriage or the
Facts: Issue: husband’s acquisition of citizenship, as the case may
On June 22, 1958 Felisa Lee, a Chinese citizen, married Whether an alien woman who marries a Filipino citizen be, for the truth is that the same situation obtains even as
Jackson Barra, a Filipino citizen. Claiming to have should undergo naturalization proceeding in order to to native-born Filipinos.
acquired the citizenship of her husband by virtue of her acquire the citizenship of her husband
marriage on the ground that she possessed all the Everytime the citizenship of a person is material or
qualifications and none of the disqualifications for Ruling: indispensable in a judicial or administrative case,
naturalization as a Filipino citizen, she applied to the The petition of Felisa Lee is really one for declaratory whatever the corresponding court or administrative
Commissioner of Immigration for cancellation of her Alien relief, considering the prayer therein that she “be authority decides therein as to such citizenship is
Certificate of Registration. adjudged to have acquired the citizenship of her generally not considered as res adjudicata, hence it has
husband, Jackson Barra, who is a Filipino x x x.” to be threshed out again and again as the occasion may
However, the Commissioner informed her that her demand. This, as We view it, is the sense in which
petition could not be given due course by his Office “in In a long line of decisions, this Court has repeatedly held Justice Dizon referred to “appropriate proceeding” in Brito
view of its existing policy of requiring wives of Filipino that there is no proceeding established by law, or the v. Commissioner, supra. Indeed, only the good sense
citizens to go to the courts for judicial determination of rules, for the judicial declaration of the citizenship of an and judgment of those subsequently inquiring into the
whether or not they possess all the qualifications and individual. matter may make the effort easier or simpler for the
none of the disqualifications required by law to acquire persons concerned by relying somehow on the
the Filipino citizenship of their husbands.” In the case of Moy Ya Lim Yao vs. Commissioner of antecedent official findings, even if these are not really
Immigration, the SC rejected the view that an alien binding.
Accordingly, Felisa Lee filed a petition for the cancellation woman who marries a Filipino citizen should undergo
of her Alien Certificate of Registration, alleging inter alia: naturalization proceeding in order to acquire the In order to have a good starting point, the most
1. that she was a Chinese citizen affiliated with citizenship of her husband, while at the same time appropriate initial step to make is:
Nationalist China; maintaining its previous ruling that a petition for judicial The alien woman must file a petition for the cancellation
2. that she was born in Manila on March 1, 1938 declaration of her citizenship such as the remedy applied of her alien certificate of registration alleging, among
of Chinese parents; for in this case is not allowed either by law or by the other things, that she is married to a Filipino citizen and
3. that she was married to Jackson Barra, a Rules of Court, in effect sanctioned the procedure that she is not disqualified from acquiring her husband’s
Filipino citizen by birth; originally resorted to by the appellee before the citizenship pursuant to section 4 of Commonwealth Act
4. that she possessed all the qualifications to respondent Commissioner. No. 473, as amended. Upon the filing of said petition,
become a Filipino citizen by naturalization and which should be accompanied or supported by the joint
was not otherwise disqualified by law for that In the said case, the SC stated: affidavit of the petitioner and her Filipino husband to the
purpose; “The question that keeps bouncing back as a effect that the petitioner does not belong to any of the
consequence of the foregoing views is, what substitute is groups disqualified by the cited section from becoming
She claims that pursuant to the provisions of Section 15 there for naturalization proceedings to enable the alien naturalized Filipino citizen (please see attached CEB
of Commonwealth Act No. 473, she became a Filipino wife of a Philippine citizen to have the matter of her own Form 1), the Bureau of Immigration conducts an
citizen as of June 22, 1958 by reason of her marriage to citizenship settled and established so that she may not investigation and thereafter promulgates its order or
her Filipino husband; have to be called upon to prove it everytime she has to decision granting or denying the petition.”
perform an act or enter into a transaction or business or

Conflict of Laws (Private International Law) by Atty. Kristine Mae M. Quibod, EnP
JMC College of Law A.Y. 2017-2018 | Case Digest Compilation by: #TeamDreamworks Page 9
Once the Commissioner of Immigration cancels the
subject’s registration as an alien, there will probably be
less difficulty in establishing her Filipino citizenship in any
other proceeding, depending naturally on the substance
and vigor of the opposition.”

Conflict of Laws (Private International Law) by Atty. Kristine Mae M. Quibod, EnP
JMC College of Law A.Y. 2017-2018 | Case Digest Compilation by: #TeamDreamworks Page 10
G.R. No. 99358. January 30, 1995.* of the Philippines. They revoked the visa previously There is no law guaranteeing aliens married to Filipino
DJUMANTAN, petitioner, vs. HON. ANDREA D. granted to her. citizens the right to be admitted, much less to be given
DOMINGO, COMMISSIONER OF THE BOARD OF permanent residency, in the Philippines.
IMMIGRATION, HON. REGINO R. SANTIAGO and Issue:
HON. JORGE V. SARMIENTO, COMMISSIONERS Whether Djumantan’s admission into the country and the The fact of marriage by an alien to a citizen does not
BUREAU OF IMMIGRATION AND DEPORTATION, change of immigration status from a temporary visitor to withdraw her from the operation of the immigration
respondents. a permanent resident legal. laws governing the admission and exclusion of
aliens. Marriage of an alien woman to a Filipino husband
Facts: Ruling: does not ipso facto make her a Filipino citizen and does
Bernard Banez, husband of Marina Cabael, went to No. She was not lawfully admitted into the country and not excuse her from her failure to depart from the country
Indonesia as a contract worker.He then embraced and she did not lawfully acquire permanent residency. upon the expiration of her extended stay here as an
was converted to Islam. alien.
There was a blatant abuse of our immigration laws in
He then, married petitioner in accordance with Islamic effecting petitioner’s entry into the country and the Under Section 9 of the Immigration Act of 1940, it is not
rites. Banez then returned to the Philippines. Petitioner change of her immigration status from temporary visitor mandatory for the CID to admit any alien who applies for
and her two children with Banez arrived in Manila as the to permanent resident. All such privileges were obtained a visitor’s visa. Once admitted into the country, the alien
“guests” of Banez. The latter made it appear that he was through misinterpretation. Never was the marriage of has no right to an indefinite stay. Under Section 13 of the
just a friend of the family of petitioner and was merely petitioner to Banez disclosed to the immigration law, an alien allowed to stay temporarily may apply for a
repaying the hospitality extended to him during his stay in authorities in her applications for temporary visitor’s visa change of status and “may be admitted” as a permanent
Indonesia. Banez executed an “Affidavit of Guaranty and and for permanent residency. resident. Among those considered qualified to apply for
Support,” for his “guests.” As “guests,” petitioner and her permanent residency if the wife or husband of a
two children lived in the house of Banez. Petitioner and The civil status of an alien applicant for admission as a Philippine citizen. The entry of aliens into the country and
her children were admitted to the Philippines as temporary visitor is a matter that could influence the their admission as immigrants is not a matter of right,
temporary visitors. exercise of discretion on the part of the immigration even if they are legally married to Filipino citizens.
authorities. The immigration authorities would be less
Marina Cabael discovered the true relationship of her inclined to allow the entry of a woman who claims to have
husband and petitioner. She filed a complaint for entered into a marriage with a Filipino citizen, who is
“concubinage”, however, subsequently dismissed for lack married to another woman.
of merit.
Generally, the right of the President to expel or deport
The immigration status of petitioner was changed from aliens whose presence is deemed inimical to the public
temporary visitor to that of permanent resident. Petitioner interest is as absolute and unqualified as the right to
was issued an alien certificate of registration. prohibit and prevent their entry into the country. This right
is based on the fact that since the aliens are not part of
Banez’ eldest son, Leonardo, filed a letter complaint the nation, their admission into the territory is a matter of
subsequently referred to Commision on Immigration and pure permission and simple tolerance which creates no
Deportation (CID). Petitioner was detained at the CID obligation on the part of the government to permit them to
detention cell. Petitioner moved for the dismissal of the stay.
deportation case on the ground that she was validly The interest, which an alien has in being admitted into or
married to a Filipino citizen. CID disposed that the allowed to continue to reside in the country, is protected
second marriage of Bernardo Banes to respondent only so far as Congress may choose to protect it.
Djumantan irregular and not in accordance with the laws

Conflict of Laws (Private International Law) by Atty. Kristine Mae M. Quibod, EnP
JMC College of Law A.Y. 2017-2018 | Case Digest Compilation by: #TeamDreamworks Page 11
No. L-27429. August 27, 1969. Ruling: and fidelity to any foreign prince, potentate" and
IN THE MATTER OF THE PETITION FOR ADMISSION First issue: particularly to the state "of which" he is "a subject or
AS CITIZEN OF THE PHILIPPINES. OH HEK HOW, No. citizen." The obvious purpose of this requirement is to
petitioner-appellee, vs. REPUBLIC OF THE At the outset, it is obvious that the oath of allegiance divest him of his former nationality, before acquiring
PHILIPPINES, oppositor-appellant. taken by petitioner on November 28, 1966, and the Philippine citizenship, because, otherwise, he would have
certificate of naturalization issued to him in pursuance two nationalities and owe allegiance to two (2) distinct
Doctrine: thereof, as well as the authority given therefor by the sovereignties, which our laws do not permit, except that,
The question of how a Chinese citizen may strip himself lower court, are null and void. pursuant to Republic Act No. 2639, "the acquisition of
of that status is governed—pursuant to Articles 15 and citizenship by a natural-born Filipino citizen from one of
1.6 of our Civil Code—by the national law of the The order of February 9, 1966 (oath-taking) had not — the Iberian and any friendly democratic Ibero-American
applicant, by the laws of China not by those of the and up to the present has not become final and countries shall not produce loss or forfeiture of his
Philippines. executory in view of the appeal duly taken by the Philippine citizenship, if the law of that country grants the
Government. same privilege to its citizens and such had been agreed
Facts: upon by treaty between the Philippines and the foreign
On January 16, 1964, a decision granting petition for What is more, petitioner's second oath was taken, not country from which citizenship is acquired."
naturalization was granted to Oh Hek How. Thus, having only after the filing of the notice of appeal and the
been granted such, he filed a motion alleging that he had submission of the record on appeal, but also after the
complied with the requirements of Republic Act No. 530 approval thereof, In other words. the lower court had
and praying that he be allowed to take his oath of already lost its jurisdiction over the case.
allegiance as such citizen and be issued the
corresponding certificate of naturalization. Second Issue:
Yes.
On February 9, 1966, the Court of First Instance of
Zamboanga del Norte issued forthwith an order It is argued that the permission is not required by our
authorizing the taking of said oath. On that same date, laws and that the naturalization of an alien, as a citizen of
petitioner took it and the certificate of naturalization was the Philippines, is governed exclusively by such laws and
issued to him. cannot be controlled by any foreign law.

The Government seasonably gave notice of its intention. However, the question of how a Chinese citizen may strip
to appeal from said order of February 9, 1966 and filed himself of that status is necessarily governed —pursuant
Its record on appeal. Before the same was approved, it to Articles 15 and 16 of our Civil Code — by the laws of
also moved to cancel petitioner's certificate of China, not by those of the Philippines. As a
naturalization, upon the ground, among others, that it consequence, a Chinese national cannot be naturalized
was issued and the oath taken before said order of as a citizen of the Philippines, unless he has complied
February 9, 1966, had become final and executory. with the laws of Nationalist China requiring previous
permission of its Minister of the Interior for the
Issue: renunciation of nationality.
(1) Whether the oath valid
(2) Whether a permission to renounce citizenship is Section 12 of Commonwealth Act No.473 provides,
necessary from the Minister of the Interior of however, that before the naturalization certificate is
Nationalist China. issued, the petitioner shall "solemnly swear," interalia,
that he renounces "absolutely and forever all allegiance

Conflict of Laws (Private International Law) by Atty. Kristine Mae M. Quibod, EnP
JMC College of Law A.Y. 2017-2018 | Case Digest Compilation by: #TeamDreamworks Page 12
G.R. No. 61565. August 20,1990.* therefore, imbued with the latter's Philippine appropriate adversary proceeding. As a matter of fact,
REPUBLIC OF THE PHILIPPINES, petitioner, vs. HON. nationality. the opposition of the Solicitor General dated February 20,
SOFRONIO SAYO, Presiding Judge of the CFI of 1970 while questioning the use of Article 412 of the Civil
Nueva Vizcaya, Branch I, and RAMON TAN BIANA, By its decision, the trial court ordered the Local Civil Code in relation to Rule 108 of the Revised Rules of
JR., respondents. Registrar of Solano, Nueva Vizcaya, to make in the Court admits that 'the entries sought to be corrected
entries and records of this office the corrections sought should be threshed out in an appropriate proceeding.'
by private respondent so as to reflect a change in the
Facts: citizenship of Ramon Tan Biana, Jr. as well as his The persons who must be made parties to a proceeding
Ramon Tan Biana, Jr. was born in Solano, Nueva parents Ramon Tan Biana and Tiu Muy from "Chinese" to concerning the cancellation or correction of an entry in
Vizcaya, as the fifth legitimate child of the spouses "Filipino" and to furnish copies of the corrected Certificate the civil register are—
Ramon Tan Biana and Tiu Muy. His birth was registered of Birth to "all other offices concerned." (1) the civil registrar, and
on the same day in the Office of the Local Civil Registrar (2) all persons who have or claim any interest which
of Solano, Nueva Vizcaya, by the nurse who attended the The petitioner however, alleged that the trial court should would be affected thereby.
parturition of Ramon's mother. Respondent (Ramon) not have ordered the correction of the relevant entries in
claims that, in the process, the attending nurse the Local Civil Registry since they involved substantial Upon the filing of the petition, it becomes the duty of the
erroneously reported to the Local Civil Registrar that matters which should not have been decided in "a merely court to—
Ramon's citizenship, and the citizenship of his parents, summary proceeding" but rather in "an appropriate action (1) issue an order fixing the time and place for the
as "Chinese" instead of "Filipino". Private respondent wherein all parties who may be affected by the entries hearing of the petition, and
claims that his "true and real citizenship", and that of his are notified or represented." (2) cause the order for hearing to be published once a
parents, is Philippine citizenship. week for three (3) consecutive weeks in a newspaper of
Issue: general circulation in the province.
Respondent filed a petition before the CFI of Nueva Whether correction on the relevant entries in the birth
Vizcaya. In this petition, private respondent sought the certificate, as that presented in the present case, may be The following are likewise entitled to oppose the
correction of entries in the Civil Registry of Solano, corrected through a summary hearing petition;—
Nueva Vizcaya, relating to his citizenship and the (1) the civil registrar, and
citizenship of his legitimate parents, as appearing in his Ruling: (2) any person having or claiming any interest under the
Certificate of Birth. The Supreme Court in this case affirmed the decision of entry whose cancellation or correction is sought.
the trial court.
Evidences presented: If all these procedural requirements have been followed,
(1) a decision of the Bureau of Immigration in I.C. In Republic of the Philippines v. Leonor Valencia, the SC a petition for correction and/or cancellation of entries in
No. 2480 dated June 18, 1947, stating: held, "It is undoubtedly true that if the subject matter of a the record of birth even if filed and conducted under Rule
'IN VIEW THEREOF, this Board is of the petition is not for the correction of clerical errors of a 108 of the Revised Rules of Court can no longer be
opinion, and so holds that the herein applicant, harmless and innocuous nature, but one involving described as 'summary'. There can be no doubt that
RAMON TAN BIANA, being a natural son of nationality or citizenship, which is indisputably substantial when an opposition to the petition is filed either by the
Marciana Goco, a Filipino woman, is entitled to as well as controverted, affirmative relief cannot be Civil Registrar or any person having or claiming any
readmission into this country as a Filipino granted in a proceeding summary in nature. However, it interest in the entries sought to be cancelled and/or
citizen.' is also true that a right in law may be enforced and a corrected and the opposition is actively prosecuted, the
(2) Another decision of the Bureau of Immigration in wrong may be remedied as long as the appropriate proceedings thereon become adversary proceedings."
I.C. No. 12890-C dated April 4, 1956, xxx As a remedy is used. This Court adheres to the principle
consequence of this marriage, which is in full that even substantial errors in a civil registry may be
force and effect, it cannot be denied that TIU corrected and the true facts established provided the Evidences presented before the trial court
MUY is the legal wife of Ramon Tan Biana, and parties aggrieved by the error avail themselves of the

Conflict of Laws (Private International Law) by Atty. Kristine Mae M. Quibod, EnP
JMC College of Law A.Y. 2017-2018 | Case Digest Compilation by: #TeamDreamworks Page 13
It remains only to note that the evidence submitted by
private respondent before the trial court consisted of the
following:
1. The Voter's Identification Card of his father, Ramon
Tan Biana, Sr. xx Goco, showing his father to be a
registered voter;
2. A decision of the Board of Special Inquiry of the
Bureau of Immigration in I.C. No. 2480, dated 18 June
1947 stating that private respondent's father being a
natural son of Marciana Goco, a Filipino citizen, was
entitled to re-admission into the country as a Philippine
citizen;
3. A certification issued by the Bureau of Immigration
stating that Ramon Tan Biana, Sr. is a Philippine citizen;
4. The decision of the Bureau of Immigration in I.C. No.
12890-C dated 4 April 1956 stating that private
respondent's mother, Tiu Muy, being the legal wife of
Ramon Tan Biana, was also a Philippine citizen;
5. The Birth Certificate of Alfred Tan, a brother of private
respondent, stating that he is the sixth child of Ramon
Tan Biana, Sr. and Tiu Muy, who are both Philippine
citizens; and
6. The Voter's Identification Card of private respondent
showing him to be a registered voter in the Philippines.

Conflict of Laws (Private International Law) by Atty. Kristine Mae M. Quibod, EnP
JMC College of Law A.Y. 2017-2018 | Case Digest Compilation by: #TeamDreamworks Page 14
G.R. No. L-32181 March 5, 1986 substantial changes in the civil registry records involving granted in a proceeding summary in nature. However, it
REPUBLIC OF THE PHILIPPINES, petitioner, the civil status of parents, their nationality or citizenship is also true that a right in law may be enforced and a
vs. may be allowed. wrong may be remedied as long as the appropriate
LEONOR VALENCIA, as Natural mother and guardian remedy is used. This Court adheres to the principle that
of her minor children, BERNARDO GO and JESSICA [[may not read]] (continuation above) if- (1) the even substantial errors in a civil registry may be
GO; and THE HON. AGAPITO HONTANOSAS, Judge proper suit is filed, and (2) evidence is submitted, either corrected and the true facts established provided the
of the COURT OF FIRST INSTANCE OF CEBU, to support the allegations of the petition or to disprove the parties aggrieved by the error avail themselves of the
Branch XI. same; that respondents have complied with these appropriate adversary proceeding.
GUTIERREZ, JR., J.: /Mishing Alaba requirements by filing the present special proceeding for
cancellation or correction of entries in the civil registry As a matter of fact, just three years after the Ty Kong Tin
[Filipino citizens ang minors, see Other information for pursuant to Rule 108 of the Revised Rules of Court and decision, this Honorable Court allowed a party to
more details :D] that they have caused reasonable notice to be given to correct mistakes involving such substantial matters
the persons named in the petition and have also caused as his birthplace and citizenship in the birth
FACTS: Respondent Leonor Valencia, for and in behalf the order for the hearings of their petition to be published certificates of his two sons. (Lim v. Republic, No. L-
of her minor children, Bernardo Go and Jessica Go filed for three (3) consecutive weeks in a newspaper of 8932, May 31, 1957, 101 Phil. 1235)
a petition for the cancellation and/or correction of entries general circulation in the province. Only that where the correction pertains to matters which
of birth of her children in the Civil Registry of the City of are important and controversial certain conditions sine
Cebu. The Local Civil Registrar of Cebu City filed a motion to que non have to be complied with. Thus it was held:
dismiss on the ground that since the petition seeks to If it refers to a substantial change which affects the
The SG opposed it alleging that the SC ruled in Ty Kong change the nationality or citizenship of Bernardo Go and status or citizenship of a party, the matter should
Tin v. Republic that the petition for correction of entry in Jessica Go from "Chinese" to "Filipino" and their status be threshed out in a proper action ... .' (Ty Kong
the Civil Registry pursuant to Article 412 of the NCC in from "Legitimate" to Illegitimate", and changing also the Tin v. Republic, supra)
relation to Rule 108 of the ROC, contemplates a status of the mother from "married" to "single" the . . . . for changes involving the civil status of the
summary proceeding and correction of mere clerical corrections sought are not merely clerical but substantial, parents, their nationality or citizenship, those are
errors, those harmless and innocuous changes such as involving as they do the citizenship and status of the grave and important matters which may have a
the correction of a name that is merely mispelled, petitioning minors and the status of their mother. bearing and effect on the citizenship and
occupation of parents, etc., and not changes or nationality not only of said parents, but of the
corrections involving civil status, nationality, or citizenship RTC: denied the motion to dismiss. It ordered the offsprings, and to seek said changes, it is not only
which are substantial and controversial. correction of the petitioner's citizenship and civil status the State, but also all parties concerned and
and the citizenship and civil status of her minor children affected should be made parties defendants or
[[may not read : NOTE: The petitioner premises Bernardo Go and Jessica Go respondents, and evidence should be submitted,
its case on precedents from the 1954 case of Ty either to support the allegations of the petition or
Kong Tin v. Republic to the 1981 case of ISSUE: Whether or not substantial errors in a civil complaint, or also to disprove the same so that
Republic v. Caparosso, that entries which can registry may be corrected any order or decision in the case may be made in
be corrected under Article 412 of the NCC as the entry in a civil register that will affect or even
implemented by Rule 108 of the ROC refer to RULING: YES. determine conclusively the citizenship or
those mistakes that are clerical in nature or nationality of a person therein involved.
changes that are harmless and innocuous]] It is undoubtedly true that if the subject matter of a
petition is not for the correction of clerical errors of a If at all what is forbidden is, in the words of Mr.
Valencia admitted that the present petition seeks harmless and innocuous nature, but one involving Justice J.B.L. Reyes, 'only the entering of material
substantial changes involving the civil status and nationality or citizenship, which is indisputably substantial corrections or amendments in the record of birth
nationality or citizenship of respondents, but alleged that as well as controverted, affirmative relief cannot be by virtue of a judgment in a summary action

Conflict of Laws (Private International Law) by Atty. Kristine Mae M. Quibod, EnP
JMC College of Law A.Y. 2017-2018 | Case Digest Compilation by: #TeamDreamworks Page 15
against the Civil Registrar. (Matias v. Republic, Civil Registrar or any person having or claiming any
No. L-26982, May 8, 1969. interest in the entries sought to be cancelled and/or The sisters and brother are:
corrected and the opposition is actively prosecuted, the 1. Sally Go, born on April 29, 1934 was licensed as a
It will thus be gleaned from the foregoing that proceedings thereon become adversary proceedings. Pharmacist after passing the government board
corrections involving such matters as the civil examinations in 1956.
status of the parents, their nationality or In the instant case, a petition for cancellation and/or 2. Fanny Go, born on July 12, 1936 is a Registered
citizenship may be allowed provided the correction of entries of birth of Bernardo Go and Jessica Nurse who passed the government board examinations
proper suit is filed. Go in the Civil Registry of the City of Cebu was filed by in 1960.
respondent Leonor Valencia on January 27, 1970, and 3. Corazon Go, born on June 20, 1939, during the trial of
[spec pro topic] OTHER ISSUE: Whether or not the pursuant to the order of the trial court dated February 4, this case in 1970 was a fourth year medical student,
proper suit or appropriate action was filed by Valencia 1970, the said petition was published once a week for qualified to take the government board examinations
three (3) consecutive weeks in the, Cebu Advocate, a after successfully completing the requirements for a
RULING: YES. The Court is of the opinion that the newspaper of general circulation in the City of Cebu. career in medicine, and presumably is a licensed
petition filed by the respondent in the lower court by way Notice thereof was duly served on the Solicitor General. physician now.
of a special proceeding for cancellation and/or correction the Local Civil Registrar and Go Eng. The order likewise 4. Antonio Go, born February 14, 1942 was an
of entries in the civil register with the requisite notice and set the case for hearing and directed the local civil engineering student during the 1970 trial of the case and
publication and the recorded proceedings that actually registrar and the other respondents or any person qualified by citizenship to take government board
took place thereafter could very well be regarded as that claiming any interest under the entries whose corrections examinations.
proper suit or appropriate action. were sought, to file their opposition to the said petition. 5. Remedios Go, born October 4, 1945 was a licensed
An opposition to the petition was consequently filed by Optometrist after passing the government board
The persons who must be made parties to a proceeding the Republic on February 26, 1970. Thereafter a full- examinations in 1967.
concerning the cancellation or correction of an entry in blown trial followed with respondent Leonor Valencia
the civil register are-(1) the civil registrar, and (2) all testifying and presenting her documentary evidence in The above facts were developed and proved during trial.
persons who have or claim any interest which would be support of her petition. The Republic on the other hand The Republic failed to refute the citizenship of the minors
affected thereby. Upon the filing of the petition, it cross-examined respondent Leonor Valencia. Bernardo and Jessica Go.
becomes the duty of the court to-(l) issue an order fixing
the time and place for the hearing of the petition, and (2) To follow the petitioner's argument that Rule 108 is not In this petition, it limits itself to a procedural reason to
cause the order for hearing to be published once a week an appropriate proceeding without in any way intimating overcome substantive findings by arguing that the proper
for three (3) consecutive weeks in a newspaper of what is the correct proceeding or if such a proceeding procedure was not followed.
general circulation in the province. exists at all, would result in manifest injustice.
Leonor Valencia is a registered voter and had always
The following are likewise entitled to oppose the petition: OTHER INFORMATION: exercised her right of suffrage from the time she reached
(I) the civil registrar, and (2) any person having or Apart from Bernardo Go and Jessica Go, there are four voting age until the national elections immediately
claiming any interest under the entry whose cancellation (4) other sisters and one (1) other brother born of the preceding the filing of her petition. The five other sisters
or correction is sought. same father and mother. Not only are all five registered and brother are also registered voters and likewise
as Filipino citizens but they have pursued careers which exercised the right of suffrage.
If all these procedural requirements have been followed, require Philippine citizenship as a mandatory pre-
a petition for correction and/or cancellation of entries in requisite. To emphasize the strict policy of the An uncle of the mother's side had held positions in the
the record of birth even if filed and conducted under Rule government regarding professional examinations, it was government having been elected twice as councilor and
108 of the Revised Rules of Court can no longer be the law until recently that to take the board exams for twice as vice-mayor of Victorias, Negros Occidental.
described as "summary". There can be no doubt that pharmacist, the applicant should possess natural born Respondent Leonor Valencia has purchased and
when an opposition to the petition is filed either by the citizenship. registered two (2) parcels of land

Conflict of Laws (Private International Law) by Atty. Kristine Mae M. Quibod, EnP
JMC College of Law A.Y. 2017-2018 | Case Digest Compilation by: #TeamDreamworks Page 16
The right of suffrage is one of the important rights of a
citizen. This is also true with respect to the acquisition of
a real property. The evidence further shows that her
children had been allowed to take the Board
Examinations given by the Government for Filipino
citizens only.

It would be a denial of substantive justice if two


children proved by the facts to be Philippine citizens,
and whose five sisters and brother born of the same
mother and father enjoy all the rights of citizens, are
denied the same rights on the simple argument that
the "correct procedure" not specified or even
intimated has not been followed.

Conflict of Laws (Private International Law) by Atty. Kristine Mae M. Quibod, EnP
JMC College of Law A.Y. 2017-2018 | Case Digest Compilation by: #TeamDreamworks Page 17
G.R. No. L-24530 October 31, 1968 RTC: After trial, the Court rendered judgment finding In this appeal, petitioners maintain that respondents were
BOARD OF IMMIGRATION COMMISSIONERS and that "the petitioners herein are the illegitimate children Filipino citizens when they left the Philippines in 1946,
COMMISSIONER OF IMMIGRATION, petitioners, of Emilia Callano, a Filipino citizen, with her they lost that citizenship, firstly, by staying in China for a
vs. common-law husband — a Chinese citizen," and period of fifteen years, and secondly, because they were
BEATO GO CALLANO, MANUEL GO CALLANO, concluding that "until the petitioners left for China in recognized by their common-law father, they became
GONZALO GO CALLANO, JULIO GO CALLANO and 1947, they must be considered as citizens of the citizens of the Republic of China in accordance with the
THE COURT OF APPEALS, respondents. Philippines as they were born of a Filipino mother and an Chinese Nationality Law.
DIZON, J.: /Mishing Alaba alien father who, however, was not married to their
mother." ISSUE: Whether or not petitioners are Filipino citizens?
Doctrine: (1) A protracted stay in a foreign country does
not amount to renunciation. (2) Recognition of the Notwithstanding the above finding and conclusion, RULING: YES. The question, whether petitioners who
respondents by their alien father is not among the ground however, the Court dismissed the case [[holding that are admittedly Filipino citizens at birth subsequently
for losing Philippine citizenship under Philippine law, and "the petitioners are citizens of the Republic of China and acquired Chinese citizenship under the Chinese Law of
it cannot be said that the respondents lost their former not being properly documented for entry into the Nationality by reason of recognition or a prolonged stay
status by reason of such recognition. Philippines as found by the Immigration Commissioner, in China, is a fit subject for the Chinese law and the
the writ of preliminary injunction heretofore issued by this Chinese court to determine, which cannot be resolved by
FACTS: ✔ The Board of Special Inquiry admitted Beato Court shall be deemed dissolved upon finality of this a Philippine court without encroaching on the legal
Callano and his three brothers for entry as citizens. decision."]] The grounds upon which the Court based system of China.
its decision were: (1) because petitioners stayed in
✔ The Board of Immigration Commissioners issued an China for a period of 15 years before returning to the For, the settled rule of international law, affirmed by the
order reversing it. It ordered their exclusion as aliens not Philippines, they must be considered as citizens of the Hague Convention on Conflict of Nationality Laws of April
properly documented for admission pursuant to Section Chinese Republic; (2) as petitioners were recognized by 12, 1930 and by the International Court of Justice, is that
27 (a) (17) of the Philippine Immigration Act of 1940, and their alien father as his children, they became Chinese "Any question as to whether a person possesses the
ordered that they be returned to the port whence they citizens under the Chinese law of nationality. While the nationality of a particular state should be determined
came or to the country of which they were nationals, Court also found that the cable authorization mentioned in accordance with laws of that state. [[There was no
upon the ground that they had been able "to enter this heretofore was a forgery, it held that, for the purpose of necessity of deciding that question because so far as
country and gain admission as Filipino citizens by the the petition before it, "it was immaterial to determine the concerns the petitioners' status, the only question in this
fraudulently secured authorization." genuineness or falsity of the cable authorization. For if proceeding is: Did the petitioners lose their Philippine
the petitioners are Filipino citizens, they are entitled to citizenship upon the performance of certain acts or the
The Commissioner of Immigration issued a warrant of remain within the territorial jurisdiction of the Republic in happening of certain events in China? In deciding this
exclusion commanding the deportation officer "to carry whatever way they might have entered." question no foreign law can be applied.]]
out the exclusion of the Callano brothers.
After the denial of herein respondents' MR, they The petitioners are admittedly Filipino citizens at
Callano brothers filed an action for injunction to restrain appealed to the CA where they raised the following birth, and their status must be governed by
the petitioners from executing the order of exclusion or issues: (a) that being Filipino citizens by birth, they did Philippine law wherever they may be, in conformity with
deportation already mentioned. They based their action not lose their citizenship nor acquire Chinese citizenship, Article 15 of the Civil Code which provides as follows:
on the ground that the Board had no jurisdiction to neither by their prolonged stay in China nor by their "Laws relating to family rights and duties, or to the status,
exclude them from the Philippines because they were not alleged recognition by their Chinese father, and (b) that conditions and legal capacity of persons are binding upon
aliens but Filipino citizens. Months later, the CFI issued a the cablegram authorization was not a forgery. citizens of the Philippines, even though living abroad."
writ of preliminary injunction restraining the respondents Under Article IV, Section 2, of the Philippine
in the case from deporting the petitioners. CA: reversed RTC. Constitution, "Philippine citizenship may be lost or
reacquired in the manner provided by law," which implies

Conflict of Laws (Private International Law) by Atty. Kristine Mae M. Quibod, EnP
JMC College of Law A.Y. 2017-2018 | Case Digest Compilation by: #TeamDreamworks Page 18
that the question of whether a Filipino has lost his citizen and sought entry into this country, which are clear
Philippine citizenship shall be determined by no other indicia of his intent to continue his former status. The
than the Philippine law. foregoing shows that the petitioners have not lost their
Philippine citizenship.
Section 1 of Commonwealth Act No. 63, as amended
by RA No.. 106, provides that a Filipino citizen may
lose his citizenship by naturalization in a foreign
country; express renunciation of citizenship; subscribing As to the validity of cablegram (in case lang i-ask)
to an oath of allegiance to support the constitution or
laws of a foreign country; rendering service to, or Due, therefore, to the pronouncement made by the Court
accepting a commission in, the armed forces of a foreign of Appeals regarding the insufficiency of the evidence
country; cancellation of the certificate of naturalization; presented by herein petitioners to prove the alleged
declaration by competent authority that he is a deserter forgery — again, a matter not now within our power to
of the Philippine armed forces in time of war; in the case review — the questioned cablegram must be deemed to
of a woman by marriage to a foreigner if, by virtue of laws be authentic. But be that as it may, we agree with both
in force in her husband's country, she acquires his the CFI of origin and the Court of Appeals that, even
nationality. Recognition of the respondents by their assuming that said document was forged, this would not
alien father is not among the ground for losing automatically render void all the proceedings had before
Philippine citizenship under Philippine law, and it the Philippine Consulate in Hongkong and the Board of
cannot be said that the respondents lost their former Special Inquiry, both of which ended with a definite
status by reason of such recognition. About the only finding that the Callanos were Filipino citizens. That
mode of losing Philippine citizenship which closely these proceedings and finding can not be nullified by the
bears on the respondents is renunciation. Department of Foreign Affairs summarily and without
giving the parties concerned an opportunity to be heard is
But even renunciation cannot be cited in support of the too evident to require any demonstration.
conclusion that petition lost their Philippine citizenship
because the law requires an express renunciation which CHIKKA MINUTE! INFO about their life:
means a renunciation that is made known distinctly and ● Callano brothers were the illegitimate children of
explicitly and not left to inference or implication; a Go Chiao Lin, a Chinese citizen, and Emilia
renunciation manifested by direct and appropriate Callano, a Filipino citizen, who started living
language, as distinguished from that which is inferred maritally in Malitbog, Leyte, in 1934;
from conduct. ● In 1946, Go Chiao Lin, Emilia and their four
sons went to Amoy, China, on vacation, but Go
Moreover, herein petitioners were all minors when they died there the same year. In 1948, Emilia had to
were brought to China in 1946. They were without legal return to the Philippines as the maid of Consul
capacity to renounce their status. Upon their return to the Eutiquio Sta. Romana because she was
Philippines only Beato Go Callano had attained the age penniless, leaving her children behind.
of majority, but even as to him there could not have been ● Subsequently the latter were able to go to
renunciation because he did not manifest by direct and Hongkong, where they sought and obtained
appropriate language that he was disclaiming Philippine employment. In 1961, they applied with the
citizenship. On the contrary, after he has attained the age Philippine Consul General in Hongkong for entry
of majority, he applied for registration as a Philippine into the Philippines as Filipino citizens.

Conflict of Laws (Private International Law) by Atty. Kristine Mae M. Quibod, EnP
JMC College of Law A.Y. 2017-2018 | Case Digest Compilation by: #TeamDreamworks Page 19
G.R. No. L-83882 January 24, 1989 and legal capacity, after having renounced Portuguese ★ TRO issued against respondent Commissioner
IN RE PETITION FOR HABEAS CORPUS OF WILLIE citizenship upon naturalization as a Philippine citizen 22 to cease and desist from immediately deporting
YU, petitioner, resumed or reacquired his prior status as a Portuguese Yu pending the conclusion of hearings before
vs. citizen, applied for a renewal of his Portuguese passport the Board of Special Inquiry, CID (Commission
MIRIAM DEFENSOR-SANTIAGO, BIENVENIDO P. 23 and represented himself as such in official documents on Immigration and Deportation)
ALANO, JR., MAJOR PABALAN, DELEO even after he had become a naturalized Philippine ★ Respondent Commissioner filed motion to lift
HERNANDEZ, BLODDY HERNANDEZ, BENNY REYES citizen. Such resumption or reacquisition of Portuguese TRO on the basis that a summary judgment of
and JUN ESPIRITU SANTO, respondent. citizenship is grossly inconsistent with his maintenance of deportation against Yu was issued by the CID
PADILLA, J.: /Mishing Alaba Philippine citizenship. Board of Commissioners.
★ To finally dispose the case, the Supreme Court
FACTS: Petitioner Willie Yu was originally issued a This Court issued the aforementioned TRO pending ruled on the several motions and gave petitioner
Portuguese passport in 1971, valid for 5 years and hearings with the Board of Special Inquiry, CID. non-extendible 3 days within which to explain
renewed for the same period upon presentment before However, pleadings submitted before this Court after the and prove why he should still be considered a
the proper Portuguese consular officer. Despite his issuance of said TRO have unequivocally shown that citizen of the Philippines despite his acquisition
naturalization as a Philippine citizen on February 10, petitioner has expressly renounced his Philippine and use of a Portuguese passport.
1978, petitioner applied for and was issued Portuguese citizenship. The material facts are not only established by
Passport on July 21, 1981 by the Consular Section of the the pleadings — they are not disputed by petitioner. A
Portuguese Embassy in Tokyo. Said Consular Office rehearing on this point with the CID would be
certifies that his Portuguese passport expired on July 20, unnecessary and superfluous. Denial, if any, of due
1986. process was obviated when petitioner was given by the
Court the opportunity to show proof of continued
While still a citizen of the Philippines who had renounced, Philippine citizenship, but he has failed.
upon his naturalization, "absolutely and forever all
allegiance and fidelity to any foreign prince, potentate, While normally the question of whether or not a person
state or sovereignty" and pledged to "maintain true faith has renounced his Philippine citizenship should be heard
and allegiance to the Republic of the Philippines," he before a trial court of law in adversary proceedings, this
declared his nationality as Portuguese in commercial has become unnecessary as this Court, no less, upon the
documents he signed, specifically, the Companies insistence of petitioner, had to look into the facts and
registry of Tai Shun Estate Ltd. 20 filed in Hongkong satisfy itself on whether or not petitioner's claim to
sometime in April 1980. continued Philippine citizenship is meritorious.

ISSUE: Whether or not Filipino citizenship of petitioner Philippine citizenship, it must be stressed, is not a
Yu was lost by virtue of his acquisition and use of commodity or were to be displayed when required and
Portuguese passport. suppressed when convenient.

RULING: YES. The foregoing acts considered together


constitute an express renunciation of petitioner's
Philippine citizenship acquired through naturalization. In
Board of Immigration Commissioners us, Go Gallano, [[PROCEDURAL FACTS:]]
21express renunciation was held to mean a renunciation ★ Petition for habeas corpus filed for release from
that is made known distinctly and explicitly and not left to detention of petitioner Yu
inference or implication. Petitioner, with full knowledge, ★ Petition denied

Conflict of Laws (Private International Law) by Atty. Kristine Mae M. Quibod, EnP
JMC College of Law A.Y. 2017-2018 | Case Digest Compilation by: #TeamDreamworks Page 20
G.R. No. 151914 July 31, 2002 petitioner was voted for and received the highest number citizenship, petitioner was an alien without any right
TEODULO M. COQUILLA, petitioner, of votes. He subsequently took his oath of office. to reside in the Philippines save as our immigration
vs. laws may have allowed him to stay as a visitor or as
THE HON. COMMISSION ON ELECTIONS and MR. On July 19, 2001, the Second Division of the COMELEC a resident alien.
NEIL M. ALVAREZ, respondents. granted private respondent’s petition and ordered the
MENDOZA, J.: /Mishing Alaba cancellation of petitioner’s certificate of candidacy. In Caasi v. Court of Appeals,25 this Court ruled that
immigration to the United States by virtue of a
FACTS: Petitioner Coquilla was born on February 17, ISSUE: Whether or not COMELEC’s order of cancellation "greencard," which entitles one to reside
1938 of Filipino parents in Oras, Eastern Samar. He grew of Coquilla’s COC was proper permanently in that country, constitutes
up and resided there until 1965, when he joined the US abandonment of domicile in the Philippines. With
Navy. He was subsequently naturalized as a U.S. RULING: Yes, it was proper. more reason then does naturalization in a foreign
citizen.2 From 1970 to 1973, petitioner thrice visited the country result in an abandonment of domicile in the
Philippines while on leave from the U.S. Navy. Even after [[for reference only]] Section 39(a) of the Local Philippines.
his retirement from the U.S. Navy in 1985, he remained Government Code (R.A No. 7160) provides:
in the US. Coquilla cannot contend that he was "compelled to adopt
Qualifications. - (a) An elective local official must be a American citizenship" only by reason of his service in the
On October 15, 1998, petitioner came to the Philippines citizen of the Philippines; a registered voter in the U.S. armed forces.26 It is noteworthy that petitioner was
and took out a residence certificate, although he barangay, municipality, city, or province or, in the case of repatriated not under R.A. No. 2630, which applies to the
continued making several trips to the US. Subsequently, a member of the sangguniang panlalawigan, repatriation of those who lost their Philippine citizenship
petitioner applied for repatriation under R.A. No. 8171 sangguniang panlungsod, or sangguniang bayan, the by accepting commission in the Armed Forces of the
which was approved on November 7, 2000, and, on district where he intends to be elected; a resident therein United States, but under R.A. No. 8171, which, as earlier
November 10, 2000, he took his oath as a citizen of the for at least one (1) year immediately preceding the day of mentioned, provides for the repatriation of, among others,
Philippines. the election; and able to read and write Filipino or any natural-born Filipinos who lost their citizenship on
other local language or dialect. (Emphasis added) account of political or economic necessity. In any event,
On November 21, 2000, petitioner applied for registration the fact is that, by having been naturalized abroad,
as a voter of Butnga, Oras, Eastern Samar. On February [[may not read if not asked]] he lost his Philippine citizenship and with it his
27, 2001, he filed his certificate of candidacy stating The term "residence" is to be understood not in its residence in the Philippines. Until his reacquisition of
therein that he had been a resident of Oras, Eastern common acceptation as referring to "dwelling" or Philippine citizenship on November 10, 2000,
Samar for "two (2) years."7 "habitation,"21but rather to "domicile" or legal residence,22 petitioner did not reacquire his legal residence in this
that is, "the place where a party actually or constructively country.
Respondent Neil M. Alvarez, who was the incumbent has his permanent home, where he, no matter where he
mayor of Oras and who was running for reelection, may be found at any given time, eventually intends to OTHER ISSUE: Whether the COMELEC retained
sought the cancellation of petitioner’s certificate of return and remain (animus manendi)."23 A domicile of jurisdiction to decide this case notwithstanding the
candidacy on the ground that the latter had made a origin is acquired by every person at birth. It is usually the proclamation of petitioner – YES
material misrepresentation in his certificate of candidacy place where the child’s parents reside and continues until
by stating that he had been a resident of Oras for two the same is abandoned by acquisition of new domicile Did the COMELEC thereby lose authority to act on the
years when in truth he had resided therein for only about (domicile of choice). petition filed by private respondent? - NO
six months since November 10, 2000, when he took his
oath as a citizen of the Philippines. In the case at bar, Coquilla lost his domicile of origin RULING: COMELEC still had jurisdiction over his case
in Oras by becoming a U.S. citizen after enlisting in although he was already proclaimed because RA 6646
The COMELEC was unable to render judgment on the the U.S. Navy in 1965. From then on and until provides:
case before the elections on May 14, 2001. Meanwhile, November 10, 2000, when he reacquired Philippine

Conflict of Laws (Private International Law) by Atty. Kristine Mae M. Quibod, EnP
JMC College of Law A.Y. 2017-2018 | Case Digest Compilation by: #TeamDreamworks Page 21
SECTION 6. Effect of Disqualification Case. – citizenship, or at the same time when one acquires
Any candidate who has been declared by final Philippine citizenship. As an alien, an individual may
judgment to be disqualified shall not be voted obtain an immigrant visa under §1328 of the Philippine
for, and the votes cast for him shall not be Immigration Act of 1948 and an Immigrant Certificate of
counted. If for any reason a candidate is not Residence (ICR)29 and thus waive his status as a non-
declared by final judgment before an election to resident. On the other hand, he may acquire Philippine
be disqualified and he is voted for and receives citizenship by naturalization under C.A. No. 473, as
the winning number of votes in such election, amended, or, if he is a former Philippine national, he may
the Court or Commission shall continue with the reacquire Philippine citizenship by repatriation or by an
trial and hearing of the action, inquiry, or protest act of Congress,30 in which case he waives not only his
and, upon motion of the complainant or any status as an alien but also his status as a non-resident
intervenor, may during the pendency thereof alien.
order the suspension of the proclamation of
such candidate whenever the evidence of his In the case at bar, the only evidence of petitioner’s status
guilt is strong. (Emphasis added) when he entered the country on October 15, 1998,
December 20, 1998, October 16, 1999, and June 23,
The rule then is that candidates who are disqualified by 2000 is the statement "Philippine Immigration [–]
final judgment before the election shall not be voted for Balikbayan" in his 1998-2008 U.S. passport. As for his
and the votes cast for them shall not be counted. But entry on August 5, 2000, the stamp bore the added
those against whom no final judgment of disqualification inscription "good for one year stay."31 Under §2 of R.A.
had been rendered may be voted for and proclaimed, No. 6768 (An Act Instituting a Balikbayan Program), the
unless, on motion of the complainant, the COMELEC term balikbayan includes a former Filipino citizen who
suspends their proclamation because the grounds for had been naturalized in a foreign country and comes or
their disqualification or cancellation of their certificates of returns to the Philippines and, if so, he is entitled, among
candidacy are strong. Meanwhile, the proceedings for others, to a "visa-free entry to the Philippines for a period
disqualification of candidates or for the cancellation or of one (1) year" (§3(c)). It would appear then that when
denial of certificates of candidacy, which have been petitioner entered the country on the dates in question,
begun before the elections, should continue even after he did so as a visa-free balikbayan visitor whose stay as
such elections and proclamation of the winners. such was valid for one year only. Hence, petitioner can
only be held to have waived his status as an alien and as
a non-resident only on November 10, 2000 upon taking
Other contention: Petitioner contends that he his oath as a citizen of the Philippines under R.A. No.
reestablished residence in this country in 1998 when he 8171.32 He lacked the requisite residency to qualify him
came back to prepare for the mayoralty elections of Oras for the mayorship of Oras, Eastern, Samar.
by securing a Community Tax Certificate in that year and
by "constantly declaring" to his townmates of his intention
to seek repatriation and run for mayor in the May 14,
2001 elections.

SC: The status of being an alien and a non-resident can


be waived either separately, when one acquires the
status of a resident alien before acquiring Philippine

Conflict of Laws (Private International Law) by Atty. Kristine Mae M. Quibod, EnP
JMC College of Law A.Y. 2017-2018 | Case Digest Compilation by: #TeamDreamworks Page 22
CIRILO R. VALLES, petitioner, vs. COMMISSION ON COMELEC Resolutions in EPC No. 92-54 and SPA Case Australian citizenship, private respondent has effectively
ELECTIONS and ROSALIND YBASCO LOPEZ, No. 95-066, declaring her a Filipino citizen duly qualified become a stateless person and as such, is disqualified to
respondents.[G.R. No. 137000. August 9, 2000] to run for the elective position of Davao Oriental run for a public office in the Philippines; petitioner
governor. concluded.
[Dual citizen siya]
Petitioner, on the other hand, maintains that the private ISSUE:Whether or not respondent is a Filipino citizen.
SHORTER FACTS: Rosalind Ybasco Lopez was born on respondent is an Australian citizen, placing reliance on
May 16, 1934 in Australia to a Filipino father and an the admitted facts that: RULING: YES (With regard to laws governing filipino
Australian mother. In 1949, at the age of 15, she left a) In 1988, private respondent registered herself with the citizenship)
Australia and came to settle in the Philippines, where she Bureau of Immigration as an Australian national and was The Philippine law on citizenship adheres to the principle
later married a Filipino and has since then participated in issued Alien Certificate of Registration No. 404695 dated of jus sanguinis. Thereunder, a child follows the
the electoral process not only as a voter but as a September 19, 1988; nationality or citizenship of the parents regardless of the
candidate, as well. In the May 1998 elections, she ran for b) On even date, she applied for the issuance of an place of his/her birth, as opposed to the doctrine of jus
governor but Valles filed a petition for her disqualification Immigrant Certificate of Residence (ICR), and soli which determines nationality or citizenship on the
as candidate on the ground that she is an Australian. c) c) She was issued Australian Passport No. H700888 basis of place of birth.
on March 3, 1988.
FACTS: Rosalind Ybasco Lopez was born on May 16, Private respondent Rosalind Ybasco Lopez was born on
1934 in Australia to a Filipino father and an Australian Petitioner theorizes that under the aforestated facts and May 16, 1934 in Australia to Telesforo Ybasco, a Filipino
mother. In 1949, at the age of 15, she left Australia and circumstances, the private respondent had renounced citizen and Theresa Marquez, an Australian. Historically,
came to settle in the Philippines, where she later married her Filipino citizenship. He contends that in her this was a year before the 1935 Constitution took into
a Filipino and has since then participated in the electoral application for alien certificate of registration and effect and at that time, what served as the
process not only as a voter but as a candidate, as well. immigrant certificate of residence, private respondent Constitution of the Philippines were the principal
She served as Provincial Board Member of the expressly declared under oath that she was a citizen or organic acts by which the United States governed the
Sangguniang Panlalawigan of Davao Oriental. In 1992, subject of Australia; and said declaration forfeited her country. These were the Philippine Bill of July 1, 1902
1995 and 1998, when she ran for governor of Davao Philippine citizenship, and operated to disqualify her to and the Philippine Autonomy Act of August 29, 1916,
Oriental. Her election was contested by her opponents run for elective office. also known as the Jones Law.
alleging as ground therefor her alleged Australian
citizenship. As regards the COMELEC’s finding that private Among others, these laws defined who were deemed to
respondent had renounced her Australian citizenship on be citizens of the Philippine islands. The Philippine Bill of
The COMELEC ruled that private respondent Rosalind January 15, 1992 before the Department of Immigration 1902 and the Jones Law, (two organic laws) provide
Ybasco Lopez is a Filipino citizen and therefore, qualified and Ethnic Affairs of Australia and had her Australian that all inhabitants of the Philippines who were
to run for a public office because (1) her father, Telesforo passport cancelled on February 11, 1992, as certified to Spanish subjects on April 11, 1899 and resided
Ybasco, is a Filipino citizen, and by virtue of the principle by the Australian Embassy here in Manila, petitioner therein including their children are deemed to be
of jus sanguinis she was a Filipino citizen under the 1987 argues that the said acts did not automatically restore the Philippine citizens. Private respondents father,
Philippine Constitution; (2) she was married to a Filipino, status of private respondent as a Filipino citizen. Telesforo Ybasco, was born on January 5, 1879 in Daet,
thereby making her also a Filipino citizen ipso jure under According to petitioner, for the private respondent to Camarines Norte, a fact duly evidenced by a certified true
Section 4 of Commonwealth Act 473; (3) and that, she reacquire Philippine citizenship she must comply with the copy of an entry in the Registry of Births. Thus, under the
renounced her Australian citizenship on January 15, mandatory requirements for repatriation under Republic Philippine Bill of 1902 and the Jones Law, Telesforo
1992 before the Department of Immigration and Ethnic Act 8171; and the election of private respondent to public Ybasco was deemed to be a Philippine citizen. By virtue
Affairs of Australia and her Australian passport was office did not mean the restoration of her Filipino of the same laws, which were the laws in force at the
accordingly cancelled as certified to by the Australian citizenship since the private respondent was not legally time of her birth, Telesforos daughter, herein private
Embassy in Manila; and (4) furthermore, there are the repatriated. Coupled with her alleged renunciation of

Conflict of Laws (Private International Law) by Atty. Kristine Mae M. Quibod, EnP
JMC College of Law A.Y. 2017-2018 | Case Digest Compilation by: #TeamDreamworks Page 23
respondent Rosalind Ybasco Lopez, is likewise a citizen (5) By cancellation of the certificate of naturalization; (As to sec 40 RA 7160 which disqualifies persons wth
of the Philippines. (6) By having been declared by competent authority, a dual citizenship from running for any elective office)
deserter of the Philippine armed forces in time of war,
The signing into law of the 1935 Philippine Constitution unless subsequently, a plenary pardon or amnesty has
has established the principle of jus sanguinis as basis for been granted: and Petitioner also maintains that even on the assumption
the acquisition of Philippine citizenship. (7) In case of a woman, upon her marriage, to a foreigner that the private respondent had dual citizenship, still, she
if, by virtue of the laws in force in her husbands country, is disqualified to run for governor of Davao Oriental; citing
So also, the principle of jus sanguinis, which confers she acquires his nationality. Section 40 of Republic Act 7160 otherwise known as the
citizenship by virtue of blood relationship, was Local Government Code of 1991.
subsequently retained under the 1973 and 1987 In order that citizenship may be lost by renunciation,
Constitutions. Thus, the herein private respondent, such renunciation must be express. Petitioners In the aforecited case of Mercado vs. Manzano, the Court
Rosalind Ybasco Lopez, is a Filipino citizen, having been contention that the application of private respondent for clarified dual citizenship as used in the Local
born to a Filipino father. The fact of her being born in an alien certificate of registration, and her Australian Government Code and reconciled the same with Article
Australia is not tantamount to her losing her Philippine passport, is bereft of merit. IV, Section 5 of the 1987 Constitution on dual allegiance.
citizenship. If Australia follows the principle of jussoli, Recognizing situations in which a Filipino citizen may,
then at most, private respondent can also claim The mere fact that private respondent Rosalind without performing any act, and as an involuntary
Australian citizenship resulting to her possession of dual Ybasco Lopez was a holder of an Australian passport consequence of the conflicting laws of different countries,
citizenship. and had an alien certificate of registration are not be also a citizen of another state, the Court explained
acts constituting an effective renunciation of that dual citizenship as a disqualification must refer
(As to respondent’s alleged renunciation of her citizenship and do not militate against her claim of to citizens with dual allegiance.
Filipino Citizenship) Filipino citizenship. For renunciation to effectively
result inthe loss of citizenship, the same must be Thus, the fact that the private respondent had dual
Petitioner’s contention: Petitioner also contends that even express. As held by this court in the aforecited case of citizenship did not automatically disqualify her from
on the assumption that the private respondent is a Aznar, an application for an alien certificate of registration running for a public office. Furthermore, it was ruled that
Filipino citizen, she has nonetheless renounced her does not amount to an express renunciation or for candidates with dual citizenship, it is enough that
Philippine citizenship. To buttress this contention, repudiation of one's citizenship. The application of the they elect Philippine citizenship upon the filing of
petitioner cited private respondents application for an herein private respondent for an alien certificate of their certificate of candidacy, to terminate their
Alien Certificate of Registration (ACR) and Immigrant registration, and her holding of an Australian passport, as status as persons with dual citizenship. This is so
Certificate of Residence (ICR), on September 19, 1988, in the case of Mercado vs. Manzano, were mere acts of because in the certificate of candidacy, one declares that
and the issuance to her of an Australian passport on assertion of her Australian citizenship before she he/she is a Filipino citizen and that he/she will support
March 3, 1988. effectively renounced the same. Thus, at the most, and defend the Constitution of the Philippines and will
private respondent had dual citizenship - she was an maintain true faith and allegiance thereto. Such
SC: Australian and a Filipino, as well. declaration, which is under oath, operates as an effective
Under Commonwealth Act No. 63, a Filipino citizen may renunciation of foreign citizenship. Therefore, when the
lose his citizenship: Moreover, under Commonwealth Act 63, the fact that herein private respondent filed her certificate of
(1) By naturalization in a foreign country; a child of Filipino parent/s was born in another candidacy in 1992, such fact alone terminated her
(2) By express renunciation of citizenship; country has not been included as a ground for losing Australian citizenship.
(3) By subscribing to an oath of allegiance to support the ones Philippine citizenship. Since private respondent
constitution or laws of a foreign country upon attaining did not lose or renounce her Philippine citizenship,
twenty-one years of age or more; petitioners claim that respondent must go through the
(4) By accepting commission in the military, naval or air process of repatriation does not hold water.
service of a foreign country;

Conflict of Laws (Private International Law) by Atty. Kristine Mae M. Quibod, EnP
JMC College of Law A.Y. 2017-2018 | Case Digest Compilation by: #TeamDreamworks Page 24
G.R. No. 104654 June 6, 1994 Philippines and therefore disqualified from continuing to to speak and write English and any one of the principal
REPUBLIC OF THE PHILIPPINES, petitioner, vs. HON. serve as governor of the Province of Sorsogon. He is dialects; (4) that he will reside continuously in the
ROSALIO G. DE LA ROSA, PRESIDING JUDGE OF ordered to vacate his office and to surrender the same to Philippines from the date of the filing of the petition until
THE REGIONAL TRIAL COURT, BRANCH 28, MANILA the Vice-Governor of the Province of Sorsogon once this his admission to Philippine citizenship; and (5) that he
and JUAN G. FRIVALDO, respondents. decision becomes final and executory. has filed a declaration of intention or if he is excused
from said filing, the justification therefor.
FACTS: September 20, 1991 - Frivaldo filed a Private respondent, having opted to reacquire Philippine
petition for naturalization under the Commonwealth Act citizenship thru naturalization under the Revised The absence of such allegations is fatal to the petition
No. 63 before the RTC Manila. Naturalization Law, is duty bound to follow the procedure (Po Yi Bi v. Republic, 205 SCRA 400 [1992]).
prescribed by the said law. It is not for an applicant to
October 7, 1991 - Judge dela Rosa set the petition for decide for himself and to select the requirements which Likewise, the petition is not supported by the affidavit of
hearing on March 16, 1992, and directed the publication he believes, even sincerely, are applicable to his case at least two credible persons who vouched for the good
of the said order and petition in the Official Gazette and a and discard those which he believes are inconvenient or moral character of private respondent as required by
newspaper of general circulation, for 3 consecutive merely of nuisance value. The law does not distinguish Section 7 of the Revised Naturalization Law. Private
weeks, the last publication of which should be at least 6 between an applicant who was formerly a Filipino citizen respondent also failed to attach a copy of his certificate of
months before the date of the said hearing. and one who was never such a citizen. It does not arrival to the petition as required by Section 7 of the said
provide a special procedure for the reacquisition of law.
January 14, 1992 - Frivaldo asked the Judge to cancel Philippine citizenship by former Filipino citizens akin to
the March 16 hearing and move it to January 24, 1992, the repatriation of a woman who had lost her Philippine The proceedings of the trial court was marred by the
citing his intention to run for public office in the May 1992 citizenship by reason of her marriage to an alien. following irregularities: (1) the hearing of the petition
elections. Judge granted the motion and the hearing was was set ahead of the scheduled date of hearing, without
moved to February 21. No publication or copy was issued The trial court never acquired jurisdiction to hear the a publication of the order advancing the date of hearing,
about the order. petition for naturalization of private respondent. The and the petition itself; (2) the petition was heard within six
proceedings conducted, the decision rendered and the months from the last publication of the petition; (3)
February 21, 1992 - the hearing proceeded. oath of allegiance taken therein, are null and void for petitioner was allowed to take his oath of allegiance
failure to comply with the publication and posting before the finality of the judgment; and (4) petitioner took
February 27, 1992 - Judge rendered the assailed requirements under the Revised Naturalization Law. his oath of allegiance without observing the two-year
Decision and held that Frivaldo is readmitted as a citizen waiting period.
of the Republic of the Philippines by naturalization. Under Section 9 of the said law, both the petition for
naturalization and the order setting it for hearing must be A decision in a petition for naturalization becomes final
Republic of the Philippines filed a petition for Certiorari published once a week for three consecutive weeks in only after 30 days from its promulgation and, insofar as
under Rule 45 of the Revised Rules of Court in relation to the Official Gazette and a newspaper of general the Solicitor General is concerned, that period is counted
R.A. No. 5440 and Section 25 of the Interim Rules, to circulation. Compliance therewith is jurisdictional (Po Yi from the date of his receipt of the copy of the decision
annul the decision made on February 27, 1992 and to Bo v. Republic, 205 SCRA 400 [1992]). Moreover, the (Republic v. Court of First Instance of Albay, 60 SCRA
nullify the oath of allegiance taken by Frivaldo on same publication and posting of the petition and the order must 195 [1974]).
date. be in its full test for the court to acquire jurisdiction (Sy v. Section 1 of R.A. No. 530 provides that no decision
Republic, 55 SCRA 724 [1974]). granting citizenship in naturalization proceedings shall be
ISSUE: Whether or not Frivaldo was duly re-admitted to The petition for naturalization lacks several allegations executory until after two years from its promulgation in
his citizenship as a Filipino. required by Sections 2 and 6 of the Revised order to be able to observe if: (1) the applicant has left
Naturalization Law, particularly: (1) that the petitioner is the country; (2) the applicant has dedicated himself
RULING: No. The supreme court ruled that of good moral character; (2) that he resided continuously continuously to a lawful calling or profession; (3) the
Private respondent is declared NOT a citizen of the in the Philippines for at least ten years; (3) that he is able applicant has not been convicted of any offense or

Conflict of Laws (Private International Law) by Atty. Kristine Mae M. Quibod, EnP
JMC College of Law A.Y. 2017-2018 | Case Digest Compilation by: #TeamDreamworks Page 25
violation of government promulgated rules; and (4) the
applicant has committed any act prejudicial to the interest
of the country or contrary to government announced
policies.

Even discounting the provisions of R.A. No. 530, the


courts cannot implement any decision granting the
petition for naturalization before its finality.

Conflict of Laws (Private International Law) by Atty. Kristine Mae M. Quibod, EnP
JMC College of Law A.Y. 2017-2018 | Case Digest Compilation by: #TeamDreamworks Page 26
G.R. No. 86564 August 1, 1989 The first decision was penned by then COMELEC STATEMENT
RAMON L. LABO, JR., petitioner, vs. THE Chigas, Vicente Santiago, Jr., with Commissioners A) RAMON LABO, JR. Y LOZANO, date of birth 23
COMMISSION ON ELECTIONS (COMELEC) EN BANC Pabalate Savellano and Opinion concurring in full and December 1934, was married in the Philippines to an
Australian citizen. As the spouse of an Australian
AND LUIS L. LARDIZABAL, respondents Commissioner Bacungan concurring in the dismissal of
citizen, he was not required to meet normal
the petition "without prejudice to the issue of the requirements for the grant of citizenship and was
FACTS: The petitioner asks the SC to restrain respondent's citizenship being raised anew in a proper granted Australian citizenship by Sydney on 28 July
the COMELEC from looking into the question of his case." Commissioner Sagadraca reserved his vote, while 1976.
citizenship for his office as Mayor of Baguio City. Commissioner Felipe was for deferring decision until B) Any person over the age of 16 years who is granted
representations shall have been made with the Australian Australian citizenship must take an oath of allegiance
In the January 20, 1988 elections, Ramon J. Labo, Jr. Embassy for official verification of the petitioner's alleged or make an affirmation of allegiance. The wording of
was proclaimed as the new elected Mayor of Baguio City. naturalization as an Australian. the oath of affirmation is: "I ..., renouncing all other
allegiance ..." etc. This need not necessarily have any
Six days later, Luis Lardizabal, the private respondent,
effect on his former nationality as this would depend
filed a petition for quo warranto which according to the The second decision was unanimously rendered by on the citizenship laws of his former country.
petitioner may not be valid because the filing fee was not Chairman Miriam Defensor-Santiago and Commissioners C) The marriage was declared void in the Australian
paid yet. Alano and Geraldez of the Commission on Immigration Federal Court in Sydney on 27 June 1980 on the
and Deportation. It is important to observe that in the ground that the marriage had been bigamous.
While the petition for quo warranto was being argued, the proceeding before the COMELEC, there was no direct D) According to our records LABO is still an Australian
question of the petitioner's citizenship was brought about. proof that the herein petitioner had been formally citizen.
Through the records from the Australian Embassy, it was naturalized as a citizen of Australia. This conjecture, E) Should he return to Australia, LABO may face court
action in respect of Section 50 of Australian
found that the petitioner - who had married an Australian which was eventually rejected, was merely inferred from
Citizenship Act 1948 which relates to the giving of
citizen, acquired an Australian passport with the Consular the fact that he had married an Australian citizen, false or misleading information of a material nature in
ID - returned in the Philippines in 1980 for the reason that obtained an Australian passport, and registered as an respect of an application for Australian citizenship. If
his marriage became void because of bigamy. alien with the CID upon his return to this country in 1980. such a prosecution was successful, he could be
deprived of Australian citizenship under Section 21 of
It was also stated that he was granted Australian On the other hand, the decision of the CID took into the Act.
citizenship in 1976. There was also an Oath of Allegiance account the official statement of the Australian F) There are two further ways in which LABO could
taken by petitioner which renunciation of other allegiance Government dated August 12, 1984, through its Consul divest himself of Australian citizenship:
(i) He could make a declaration of Renunciation of
is included. in the Philippines, that the petitioner was still an
Australian citizenship under Section 18 of the
Australian citizen as of that date by reason of his Australian Citizenship Act, or
ISSUE: Whether or not Ramon J. Labo is a naturalization in 1976. That statement is reproduced in (ii) If he acquired another nationality, (for example,
Filipino citizen full as follows: Filipino) by a formal and voluntary act other than
I, GRAHAM COLIN WEST, Consul of Australia in the marriage, then he would automatically lose as
RULING: NO Philippines, by virtue of a certificate of appointment Australian citizenship under Section 17 of the Act.
signed and sealed by the Australian Minister of State IN WITNESS WHEREOF, I HAVE HEREUNTO SET
for Foreign Affairs on 19 October 1983, and MAY HAND AND SEAL OF THE AUSTRALIAN
There are two administrative decisions on the question of
recognized as such by Letter of Patent signed and EMBASSY, MANILA, THIS 12th DAY OF APRIL 1984.
the petitioner's citizenship. The first was rendered by the sealed by the Philippines Acting Minister of Foreign DONE AT MANILA IN THE PHILIPPINES.
Commission on Elections on May 12, 1982, and found Affairs on 23 November 1983, do hereby provide the (Signed) GRAHAM C. WEST Consul
the petitioner to be a citizen of the Philippines. The following statement in response to the subpoena
second was rendered by the Commission on Immigration Testificandum dated 9 April 1984 in regard to the This was affirmed later by the letter of February 1, 1988,
and Deportation on September 13, 1988, and held that Petition for disqualification against RAMON LABO, JR.
addressed to the private respondent by the Department
the petitioner was not a citizen of the Philippines. Y LOZANO (SPC No. 84-73), and do hereby certify
that the statement is true and correct.
of Foreign Affairs reading as follows:

Conflict of Laws (Private International Law) by Atty. Kristine Mae M. Quibod, EnP
JMC College of Law A.Y. 2017-2018 | Case Digest Compilation by: #TeamDreamworks Page 27
that he was a citizen of Australia in a number of sworn The petitioner now claims that his naturalization in
Sir: statements voluntarily made by him and. even sought to Australia made him at worst only a dual national and did
With reference to your letter dated 1 February 1988, I avoid the jurisdiction of the barangay court on the ground not divest him of his Philippine citizenship. Such a
wish to inform you that inquiry made with the
that he was a foreigner. specious argument cannot stand against the clear
Australian Government through the Embassy of the
Philippines in Canberra has elicited the following
provisions of CA No. 63, which enumerates the modes by
information: The decision of the COMELEC in 1982 quaintly which Philippine citizenship may be lost. Among these
1) That Mr. Ramon L. Labo, Jr. acquired Australian dismisses all these acts as "mistakes" that did not divest are: (1) naturalization in a foreign country; (2) express
citizenship on 28 July 1976. the petitioner of his citizenship, although, as earlier renunciation of citizenship; and (3) subscribing to an oath
2) That prior to 17 July 1986, a candidate for noted, not all the members joined in this finding. We of allegiance to support the Constitution or laws of a
Australian citizenship had to either swear an oath of reject this ruling as totally baseless. The petitioner is not foreign country, all of which are applicable to the
allegiance or make an affirmation of allegiance which an unlettered person who was not aware of the petitioner. It is also worth mentioning in this connection
carries a renunciation of "all other allegiance.
consequences of his acts, let alone the fact that he was that under Article IV, Section 5, of the present
Very truly yours, For the Secretary of Foreign Affairs:
(SGD) RODOLFO SEVERINO, JR. Assistant
assisted by counsel when he performed these acts. Constitution, "Dual allegiance of citizens is inimical to the
Secretary national interest and shall be dealt with by law."
The decision also noted the oath of allegiance taken There is also the claim that the decision can no longer be
by every naturalized Australian reading as follows: reversed because of the doctrine of res judicata, but this Even if it be assumed that, as the petitioner asserts, his
OATH OF ALLEGIANCE too must be dismissed. This doctrine does not apply to naturalization in Australia was annulled after it was found
I, A.B., renouncing all other allegiance, swear by questions of citizenship, as the Court has ruled in several that his marriage to the Australian citizen was bigamous,
Almighty God that I will be faithful and bear true cases. Moreover, it does not appear that it was properly that circumstance alone did not automatically restore his
allegiance to Her Majesty Elizabeth the Second,
and seasonably pleaded, in a motion to dismiss or in the Philippine citizenship. His divestiture of Australian
Queen of Australia, Her heirs and successors
according to law, and that I will faithfully observe the
answer, having been invoked only when the petitioner citizenship does not concern us here. That is a matter
laws of Australia and fulfill my duties as an Australian filed his reply to the private respondent's comment. between him and his adopted country. What we must
citizen. Besides, one of the requisites of res judicata, to wit, consider is the fact that he voluntarily and freely rejected
and the Affirmation of Allegiance, which declares: identity of parties, is not present in this case. Philippine citizenship and willingly and knowingly
AFFIRMATION OF ALLEGIANCE embraced the citizenship of a foreign country. The
I, A.B., renouncing all other allegiance, solemnly and The petitioner's contention that his marriage to an possibility that he may have been subsequently rejected
sincerely promise and declare that I will be faithful and Australian national in 1976 did not automatically divest by Australia, as he claims, does not mean that he has
bear true allegiance to Her Majesty Elizabeth the
him of Philippine citizenship is irrelevant. There is no been automatically reinstated as a citizen of the
Second, Queen of Australia, Her heirs and successors
according to law, and that I will faithfully observe the
claim or finding that he automatically ceased to be a Philippines.
Laws of Australia and fulfill my duties as an Australian Filipino because of that marriage. He became a citizen of
citizen. Australia because he was naturalized as such through a Under CA No. 63 as amended by PD No. 725, Philippine
formal and positive process, simplified in his case citizenship may be reacquired by direct act of Congress,
The petitioner does not question the authenticity of the because he was married to an Australian citizen. As a by naturalization, or by repatriation. It does not appear in
above evidence. Neither does he deny that he obtained condition for such naturalization, he formally took the the record, nor does the petitioner claim, that he has
Australian Passport No. 754705, which he used in Oath of Allegiance and/or made the Affirmation of reacquired Philippine citizenship by any of these
coming back to the Philippines in 1980, when he Allegiance, both quoted above. Renouncing all other methods. He does not point to any judicial decree of
declared before the immigration authorities that he was allegiance, he swore "to be faithful and bear true naturalization as to any statute directly conferring
an alien and registered as such under Alien Certificate of allegiance to Her Majesty Elizabeth the Second, Queen Philippine citizenship upon him. Neither has he shown
Registration No. B-323985. He later asked for the of Australia ..." and to fulfill his duties "as an Australian that he has complied with PD No. 725, providing that:
change of his status from immigrant to a returning former citizen."
Philippine citizen and was granted Immigrant Certificate ... (2) natural-born Filipinos who have lost their
of Residence No. 223809. He also categorically declared Philippine citizenship may reacquire Philippine

Conflict of Laws (Private International Law) by Atty. Kristine Mae M. Quibod, EnP
JMC College of Law A.Y. 2017-2018 | Case Digest Compilation by: #TeamDreamworks Page 28
citizenship through repatriation by applying with
the Special Committee on Naturalization created
by Letter of Instruction No. 270, and, if their
applications are approved, taking the necessary
oath of allegiance to the Republic of the
Philippines, after which they shall be deemed to
have reacquired Philippine citizenship. The
Commission on Immigration and Deportation
shall thereupon cancel their certificate of
registration.

That is why the Commission on Immigration and


Deportation rejected his application for the cancellation of
his alien certificate of registration. And that is also the
reason we must deny his present claim for recognition as
a citizen of the Philippines.

The petitioner is not now, nor was he on the day of the


local elections on January 18, 1988, a citizen of the
Philippines. In fact, he was not even a qualified voter
under the Constitution itself because of his alienage. He
was therefore ineligible as a candidate for mayor of
Baguio City, under Section 42 of the Local Government
Code.

Conflict of Laws (Private International Law) by Atty. Kristine Mae M. Quibod, EnP
JMC College of Law A.Y. 2017-2018 | Case Digest Compilation by: #TeamDreamworks Page 29
G.R. No. 105111 July 3, 1992 need to re-examine the same and make a lengthy case. Labo failed to present any evidence before the
RAMON L. LABO, Jr., petitioner, vs. COMMISSION dissertation thereon. Comelec to support his claim of reacquisition of
ON ELECTIONS, and ROBERTO ORTEGA, Philippine citizenship.
respondents.
At any rate, the fact remains that he has not submitted in Furthermore, we need only to reiterate what we have
FACTS: Ramon Labo, Jr. filed his certificate of the instant case any evidence, if there be any, to prove stated in Labo v. Comelec (supra), viz.,:
candidacy for mayor of Baguio City. Roberto Ortega, also his reacquisition of Philippine citizenship either before
filed his certificate of candidacy for the same office. this Court or the Comelec. On this score alone, no grave Under CA No. 63, as amended by P.D. No. 725,
abuse of discretion was committed by respondent Philippine citizenship may be reacquired by a direct act of
Ortega filed a disqualification proceeding against Labo Comelec in cancelling his (Labo's) certificate of Congress, by naturalization, or by repatriation. It does not
before the COMELEC on the ground that Labo made a candidacy and declaring that he is NOT a Filipino citizen appear in the record, nor does the petitioner claim, that
false representation when he stated therein that he is a pursuant to our ruling in the 1989 case of Labo v. he has reacquired Philippine citizenship by any of these
"natural-born" citizen of the Philippines. At the hearing, Comelec (supra). methods. He does not point to any judicial decree of
Ortega presented Labo v COMELEC where the SC naturalization or to any statute directly conferring
declared that Labo not a citizen of the Philippines. Petitioner Labo claims, however, that Sec. 72 of the Philippine citizenship upon him. . . .
Omnibus Election Code "operates as a legislatively
May 9, 1992 - Labo's certificate of candidacy is DENIED mandated special repatriation proceeding" and that it Petitioner Labo's status has not changed in the case at
and CANCELLED. DELETE the name of Labo from the allows his proclamation as the winning candidate since bar. To reiterate, he (Labo) was disqualified as a
list of candidates for City Mayor of Baguio City. the resolution disqualifying him was not yet final at the candidate for being an alien. His election does not
time the election was held. automatically restore his Philippine citizenship, the
May 10, 1992 - Decision disqualifying Labo shall become possession of which is an indispensable requirement for
final and executory only after 5 days from promulgation. The Court finds petitioner Labo's strained argument holding public office (Sec. 39, Local Government Code).
Labo may still continue to be voted upon as candidate for quixotic and untenable. In the first place, Sec. 72 of the Still, petitioner takes pains in raising a new argument not
City Mayor of Baguio City. Omnibus Election Code has already been repealed by litigated before the respondent Comelec. Petitioner
Sec. 6 of RA No. 6646, to wit: claims that he has reacquired his Filipino citizenship by
May 13, 1992 - SUSPEND the proclamation of Labo in Sec. 6. Effect of Disqualification Case. — Any citing his application for reacquisition of Philippine
the event he wins in the elections for the City Mayor of candidate who has been declared by final judgment to citizenship filed before the Office of the Solicitor General
Baguio. be disqualified shall not be voted for, and the votes pursuant to PD 725 and Letter of Instruction No. 270.
cast for him shall not be counted. If for any reason a
candidate is not declared by final judgment before an
ISSUE: Whether or Not Labo is a Filipino To date, however, and despite favorable
election to be disqualified and he is voted for and
citizen receives the winning number of votes in such election, recommendation by the Solicitor General, the Special
the Court or the Commission shall continue with the Committee on Naturalization had yet acted upon said
RULING: NO trial and hearing of the action, inquiry, or protest and, application for repatriation. Indeed, such fact is even
Up to this moment, petitioner Labo still failed to submit a upon motion of the complainant or any intervenor, may admitted petitioner. In the absence of any official action
scintilla of proof to shore his claim before this Court that during the pendency thereof order the suspension of or approval by the proper authorities, a mere application
he has indeed reacquired his Philippine citizenship. the proclamation of such candidate whenever the for repratriation, does not, and cannot, amount to an
evidence of his guilt is strong.
automatic reacquisition of the applicant's Philippine
Instead, petitioner relies in the US case of Vance v. citizenship.
A perusal of the above provision would readily disclose
Terrazas (supra). Suffice it to state that petitioner has
that the Comelec can legally suspend the proclamation of
already pleaded Vance in his motion for reconsideration
petitioner Labo, his reception of the winning number of
in Labo v. Comelec (supra; Rollo, p. 375). Having been
votes notwithstanding, especially so where, as in this
previously passed upon, the Court sees no pressing

Conflict of Laws (Private International Law) by Atty. Kristine Mae M. Quibod, EnP
JMC College of Law A.Y. 2017-2018 | Case Digest Compilation by: #TeamDreamworks Page 30
Fundamental Principles Governing Domicile of Choice

AGAPITO A. AQUINO vs. COMMISSION ON ELECTIONS,


MOVE MAKATI, MATEO BEDON and JUANITO ICARO; G.R.
No. 120265 September 18, 1995;

Doctrine:

Facts:

Issue:

Conflict of Laws (Private International Law) by Atty. Kristine Mae M. Quibod, EnP
JMC College of Law A.Y. 2017-2018 | Case Digest Compilation by: #TeamDreamworks Page 31
j) Marcos vs Comelec (1995)
k) Domino vs Comelec (1999)
l) Jao vs CA (2002) VINA ARANETA

Conflict of Laws (Private International Law) by Atty. Kristine Mae M. Quibod, EnP
JMC College of Law A.Y. 2017-2018 | Case Digest Compilation by: #TeamDreamworks Page 32
G.R. No. 104960 September 14, 1993 Upon receipt of the letter, Romualdez departed from the
PHILIP G. ROMUALDEZ, petitioner, U.S. for the Philippines, arriving on December 1991 "Domicile" denotes a fixed permanent residence to which
vs. apparently without any government document. when absent for business or pleasure, or for like reasons,
REGIONAL TRIAL COURT, BRANCH 7, TACLOBAN one intends to return. 20 That residence, in the case of
CITY, DONATO ADVINCULA, BOARD OF ELECTION When Romualdez arrived in the Philippines, he did not the petitioner, was established during the early 1980's to
INSPECTORS, PRECINCT No. 9, MALBOG, TOLOSA, delay his return to his residence at Malbog, Tolosa, be at Barangay Malbog, Tolosa, Leyte. Residence thus
LEYTE, and the MUNICIPAL REGISTRAR COMELEC, Leyte. During the registration of voters conducted by the, acquired, however, may be lost by adopting another
TOLOSA, LEYTE, respondents. petitioner registered himself anew as a voter at Precinct choice of domicile.
Otilia Dimayuga-Molo for petitioner. No. 9 of Malbog, Tolosa, Leyte. The chairman of the
The Solicitor General for respondents. Board of Election Inspectors, who had known Romualdez In order, in turn, to acquire a new domicile by choice,
to be a resident of the place and, in fact, an elected there must concur
FACTS: Barangay Chairman of Malbog in 1982, allowed him to be
registered. (1) residence or bodily presence in the new locality,
The petitioner is Philip Romualdez, a natural born citizen (2) an intention to remain there, and
of the Philippines, the son of the former Governor of On 21 February 1992, herein private respondent Donato (3) an intention to abandon the old domicile.
Leyte, Benjamin "Kokoy" Romualdez, and nephew of the Advincula ("Advincula") filed a petition with the Municipal
then First Lady Imelda Marcos. Sometime in the early Trial Court of Tolosa, Leyte, praying that Romualdez be In other words, there must basically be animus manendi
part of 1980, the petitioner, in consonance with his excluded from the list of voters in Precinct No. 9 of coupled with animus non revertendi. The purpose to
decision to establish his legal residence at Barangay Malbog, Tolosa, Leyte, under BP 881 and RA 7166. remain in or at the domicile of choice must be for an
Malbog, Tolosa, Leyte,1 caused the construction of his indefinite period of time; the change of residence must be
residential house therein. He soon thereafter also served Advincula alleged that Romualdez was a resident of voluntary; and the residence at the place chosen for the
as Barangay Captain of the place. In the 1984 Batasan Massachusetts, U.S.A.; that his profession and new domicile must be actual.
Election and 1986 "snap" Presidential Election, occupation was in the U.S.A.; that he had just recently
Romualdez acted as the Campaign Manager of the arrived in the Philippines; and that he did not have the The political situation brought about by the "People's
Kilusang Bagong Lipunan (KBL) in Leyte where he voted. required one-year residence in the Philippines and the Power Revolution" must have truly caused great
six-month residence in Tolosa to qualify him to register apprehension to the Romualdezes, as well as a serious
When the eventful days from the 21st to the 24th of as a voter in Barangay Malbog, Tolosa, Leyte. concern over the safety and welfare of the members of
February, 1986, came or were about to come to a close, their families. Their going into self-exile until conditions
some relatives and associates of the deposed President, ISSUE: favorable to them would have somehow stabilized is
fearing for their personal safety, whether founded or not, understandable. Certainly, their sudden departure from
"fled" the country. Petitioner Romualdez, for one, Whether or not Romualdez voluntarily left the country the country cannot be described as "voluntary," or as
together with his immediate family, left the Philippines and abandoned his residence in Malbog, Tolosa, Leyte. "abandonment of residence" at least in the context that
and sought "asylum" in the United States which the these terms are used in applying the concept of "domicile
United States (U.S.) government granted HELD: by choice."

On 25 September 1991, Romualdez received a letter No. In election cases, the Court treats domicile and We have closely examined the records, and we find not
from Mr. Charles Cobb, District Director of the U.S. residence as synonymous terms, thus: "(t)he term that much to convince us that the petitioner had, in fact,
Immigration and Naturalization Service, informing him "residence" as used in the election law is synonymous abandoned his residence in the Philippines and
that he should depart from the U.S. at his expense on or with "domicile", which imports not only an intention to established his domicile elsewhere.
before 23 August 1992 reside in a fixed place but also personal presence in that
place, coupled with conduct indicative of such intention."
19

Conflict of Laws (Private International Law) by Atty. Kristine Mae M. Quibod, EnP
JMC College of Law A.Y. 2017-2018 | Case Digest Compilation by: #TeamDreamworks Page 33
Conflict of Laws (Private International Law) by Atty. Kristine Mae M. Quibod, EnP
JMC College of Law A.Y. 2017-2018 | Case Digest Compilation by: #TeamDreamworks Page 34

Potrebbero piacerti anche